Você está na página 1de 210

EMQs by Maju Mathews

Theme: Rashes in Children


Options
A.
B.
C.
D.
E.
F.
G.
H.

Echovirus
Measles
Lyme disease
Juvenile rheumatoid disease
Rubella
Chicken pox
Meningococcemia
Erythema infectiosum

For each question below, choose the SINGLE most likely answer from the above list of
options. Each may be used once, more than once, or not at all.

1. A 6-year-old child develops fever, arthralgias and a confluent rash on the


face with a slapped check appearance.
2. A 4-year-old child presents with a generalised blanching and an
erythematous rash. He also has fever, conjunctivitis, photophobia and
coryza. Red lesions with white centers are present on the buccal mucosa.
3.

A 12-year-old girl presents with low-grade fever, postauricular


lymphadenopathy and a maculopapular rash that began on the face spread
to the trunk. She also has splenomegaly.

4. A 5-year-old is brought with purpuric rashes over his body. He has had a
fever for 3 days. He looks very ill and toxic.
5. An 8 old child is brought to the clinic with fever, malaise, and eruption of
vesicular lesion on the face, scalp and trunk. Two other children in the
school had similar complaints recently.

EMQs by Maju Mathews

Answers: Rashes in Children


1. (H)
Caused by Human parvovirus type B19. Also called fifth disease.

2. (B)
Caused by RNA Paramyxovirus. Incubation 7-12 days. Infective from first
prodromal symptom until 5 days after rash starts.

3. (E)
Caused by RNA virus. Incubation 14-21 days. Patient is infective 5 days
before and 5 days after the rash starts.

4. (G)
Is very deadly and kills swiftly. Always think about this and start treatment
before reports confirm the diagnosis.

5. (F)
Cause Varicella Zoster. Typical presentation. Incubation 11-21 days. Spreads
very rapidly.

EMQs by Maju Mathews

Theme: Management of Arrhythmias


Options
A.
B.
C.
D.
E.
F.
G.
H.
I.
J.

External pacing
Digoxin
Reassurance
Lignocaine
Amiodarone
Permanent dual chamber pacing
DC cardioversion
Flecainide
Atropine
Cardiopulmonary resuscitation

For each question below, choose the SINGLE most likely answer from the above list
of options. Each option may be used once, more than once, or not at all.

6. A 63-year-old lady following cardiac arrest underwent 10 minutes of cardiopulmonary resuscitation. The ECG now shows ventricular standstill with P
waves.
7. A 60-year-old fit and active housewife has episodes of pre-syncope. A 24hour Holter monitor shows evidence of intermittent complete heart block with
long pauses on the rhythm strips coinciding with symptoms.
8. A 75-year-old lady with a long-standing history of untreated palpitations is
now admitted with acute shortness of breath and palpitations. Examination
reveals an irregularly irregular pulse with bilateral basal crepitations and
ECG shows absent P waves with rate of 140 beats per minute.
9. A 23-year-old male presents with a history of flutter in his chest. His girl
friend has noticed his pulse to be irregular at times. An ECG shows
ventricular ectopics of 4-5 per minute. A tread mill test is unremarkable and
his ectopics disappear when his heart rate increases.
10. A 40-year-old anxious male feels faint and sweaty during a coronary
angiogram procedure. His heart rate is 35 per minute and BP is 47/55.
11. A 45-year-old male presents with a 2-hour history of sudden onset of
palpitations. ECG shows ventricular tachycardia. He is otherwise well and
haemodynamically stable.

EMQs by Maju Mathews

Answers: Management of Arrhythmias

6. (A)
Electromechanical dissociation. Sometimes termed electrical pulseless
activity.

7. (G)
Dual chamber pacemakers pace atria and ventricles sequentially, helping
stroke volumes.

8. (B)
Is used to slow ventricular rate. Dangerous in HOCM and WPW. Elderly are
at risk of toxicity. Start at low doses.

9. (C)
Asymptomatic Stable Ventricular Ectopics does not require any intervention.
ATROPINE is the drug of choice for bradycardia.

10. (J)
This is used to increase the heart rate.

11. (D) or (E)


Treatment for VF. Other drugs that can be used include Adenosine and
Procainamide. Amiodarone is the drug of choice according to the new
guidelines & had replaced lignocaine.

EMQs by Maju Mathews

Theme: Diagnosis of Chest Pain


Options
A.
B.
C.
D.
E.
F.
G.
H.
I.
J.

Unstable angina
Stable angina
Prinzmetal angina
MI
Peptic ulcer
Ankylosing spondylosis
Pleuritis
Oesophageal rupture
Gastric ulcer
Panic attack

For each question below, choose the SINGLE most likely answer from the above list of
options. Each option may be used once, more than once, or not at all.

12. A 56-year-old male under investigation for? CA stomach underwent


endoscopy yesterday. He presents to you today with severe central pain
with difficulty in breathing.
13. A 56-year-male presents with 2 episodes of central chest pain while having
his evening tea. The episode lasted for 15 minutes. He is not known to
have angina but has had two similar episodes in the past one month.
14. A 45-old abscess smoker presents with central chest pain, burning in nature
that started 30 min after lying down after a heavy meal. No radiation.
15. A 60-year-male presents to A&E anxious, sweaty and distressed with
central chest pain radiating to the neck. He points to the pain by placing a
clenched fist over his chest. BP-180/120, pulse-110/min.
16. A 32-year-old female woke up sleep with severe palpitations, sweating and
shortness of breath lasting for about 15 minutes. ECG is normal. Non
smoker and no family history of heart disease.

EMQs by Maju Mathews

Answers: Diagnosis of chest pain


12. (H)
Typical history of oesophageal rupture.

13. (A)
Stable angina is typically brought on by exertion and is usually rapidly
relieved by rest & nitrates. Unstable angina is severe or frequent angina
superimposed on chronic stable angina, angina at rest or minimal exertion or
angina of new onset brought on by minimal exertion.

14. (E)
Dyspepsia is often the presenting complaint of peptic ulcer disease.

15. (D)
Here, the patient is demonstrating Levines sign.

16. (J)
Negative symptoms and history should lead one to this diagnosis.

EMQs by Maju Mathews

Them: Diagnosis of Jaundice


Options
A.
B.
C.
D.
E.
F.
G.
H.
I.
J.
K.
L.

Chronic Active Hepatitis


Primary Sclerosing Cholangitis
Gilberts Syndrome
Gall stones
Leptospirosis
Halothane
Hemolytic anaemia
Carcinoma pancreas
Primary biliary cirrhosis
Alcoholic Hepatitis
Primary hepatocellular Carcinoma
Hemochromatosis

For each question below, choose the SINGLE most likely answer from the above list of
options. Each option may be used once, more than once, or not at all.

17. A 43 old woman has severe pruritus, pigmented deposits on her eyelids
and skin creases, and hepatosplenomegaly. She develops jaundice 4
years after the onset of her initial symptoms.
18. A 47-year-old man develops bronze pigmentation of his skin and
hepatomegaly. He had abnormal glucose tolerance test.
19. A 53-year-old man from India develops jaundice, weight loss, fever, and a
dull right hypochondrial pain. He has associated proximal myopathy.
20. A 39-year-old man develops pruritus, jaundice, abdominal pain and fever.
He is on treatment for inflammatory bowel disease.
21. A 40-year-old man presents with, jaundice, abdominal pain, high fever and
distension of the abdomen. O/E: Enlarged liver, ascites. Liver biopsy
shows Mallory bodies and giant mitochondria.

EMQs by Maju Mathews

Answers: Diagnosis of Jaundice


17. (I)
Pruritus is very common and troublesome symptom and can occur in the
absence of jaundice.

18. (L)
Clinical triad of diabetes, hepatomegaly and bronze pigmentation is seen in a
third of patients with hemochromatosis.

19. (K)
Worldwide 5%-25% of people are hepatitis B carriers and is a common cause
of hepatocellular carcinoma.

20. (B)
Primary sclerosing cholangitis occurs more commonly with ulcerative colitis.

21. (D)
Initially shows fatty change, which is reversible on withdrawal of alcohol.

EMQs by Maju Mathews

Theme: Diagnosis of Inguinal and Scrotal Swelling


Options
A.
B.
C.
D.
E.
F.
G.
H.
I.
J.
K.

Varicocele
Primary hydrocele
Secondary hydrocele
Epididymal cyst
Teratoma
Inguinal hernia
Femoral hernia
Seminoma
Torsion of testis
Idiopathic scrotal oedema
Epididymoorchitis.

For each question below, choose the SINGLE most likely answer from the above list of
options. Each option may be used once, more than once, or not at all.

22. A 20-year-old male presents with sudden onset lower abdominal & groin
pain associated with vomiting. On examination, elevation of the testis
increases the pain.
23. A 30-year-old male presenting with a two-day history of frequency &
dysuria. Has now developed pain in the testis and fever. On examination,
elevation of the testis eases the pain.
24. A 50-year-old male, presents with dragging discomfort in his right testis,
more on standing & walking. On examination, the testis feels like a bag of
worms. Cough impulse is also present.
25. A 35-year-old male presents with right testicular swelling and discomfort.
Examination reveals firm, smooth enlarged testis. No history of previous
injury to the testis.
26. A 42-year-old male noticed a swelling in his left testis two months back,
which has been progressively increasing in size. His both testis are involved
now. On examination, tense transilluminable swelling, which feels, like a tiny
bunch of grapes.

EMQs by Maju Mathews

Answers: Diagnosis of inguinal and scrotal swelling


22. (I)
Commonly presents between the ages of 10 and 25 years of age. Treat
with manipulation and early operative fixation.

23. (K)
Infection. Treat with antibiotics.

24. (A)
Various dilatation of the veins draining the testis. Operation not indicated
unless causing symptoms.

25. (H)
Staging is important before treatment. Seminomas are radiosensitive.

26. (D)
Usually found in the middle aged is often bilateral. Treatment is excision.

10

EMQs by Maju Mathews

Theme: Disease of the liver


Options
A.
B.
C.
D.
E.
F.
G.
H.
I.
J.
K.
L.

Alcoholic Liver failure


Cardiac cirrhosis
Amoebic liver abscess
Primary biliary cirrhosis
Hepatitis A infection
Haemochromatosis
Hepatocellular degeneration
Hepatocellular carcinoma
Hepatorenal syndrome
Chronic active hepatitis
Amyloidosis
1-antitrpysin deficiency

For each question below, choose the SINGLE most likely answer from the above list of
options. Each option may be used once, more than once, or not at all.

27. A 50-year-old female admitted in the surgical ITU with deterioration in her
condition. She underwent surgery for cholecystectomy & CBD stone
removal in the morning and now presents with vomiting, signs of fluid
overload and peripheral shutdown.
28. A 23-year-old with learning disabilities presents with jaundice, dysarthria
and cognitive impairment. He has also noticed jerky, purposeless hand
movements. O/E, tremors and a brownish discoloration of his sclera.
29. A 45-year-old male presents with joint pains and impotence. On
examination skin is pigmented with enlarged liver. There is a family history
of liver disease.
30. A 55-year-old female, a known case of Ulcerative Colitis, presents with
malaise, weight loss and anorexia since 3-months. She does not have
jaundice. Examination shows a hard, enlarged liver with an arterial bruit
over the liver.
31. A 35-year-old HIV positive male presents with fatigue, upper abdominal
pain, and jaundice since the past 10 months. Examination reveals clubbing,
leukonychia ascites and splenomegaly. He also has evidence of needle
tracks on his limbs.
32. A 45-year-old hirsute female presents with fatigue and pruritus. On
examination; skin pigmentation, xanthelasmas and tendon xanthomas. She
is on a gluten free diet.

11

EMQs by Maju Mathews

Answer: Diseases of the liver


27. (I)
Patients with obstructive jaundice are prone to develop renal failure after
surgery. This may be because of the toxic effect of bilirubin on the
kindness.

28. (G)
Wilsons Disease. Copper disposition in the liver and brain can cause
cirrhosis and basal ganglia destruction.

29. (F)
Autosomal recessive. Overload of iron in the body with deposition in the
liver, heart, pancreas and pituitary.

30. (H)
Chest X ray may reveal raised right hemidiaphragm.

31. (J)
Slowly progressive condition. Strong link between liver cancer and chronic
hepatitis B infection

32. (D)
Commoner in women. Pruritus is the most prominent symptom. Associated
with other autoimmune diseases.

12

EMQs by Maju Mathews

Theme: Management of Skin Conditions


Options
A.
B.
C.
D.
E.
F.
G.
H.
I.

Retinoids
Sulfonamides
Oxytetracycline
Topical Metronidazole
Malathion
PUVA
Topical Steroids
Systemic Steroids
Antihistamines

For each question below, choose the SINGLE most likely answer from the above list of
options. Each option may be used once, more than once, or not at all.

33. A 5-year-old adult develops itchy eruptions on the finger webs, buttocks and
groin.
34. An 18-year-old adult develops itchy purple papular lesions on the flexor
aspects of the ankle and the leg. Patient is a Hep-C carrier.
35. A 63-year-old with tense blisters on a reddened base
36. A 21-year-old male develops erythematous lesions on the chin and nose
with papules and pustules. It is associated with flushing with alcohol and
spicy food.
37. A 43-year-old with flaccid blisters on the skin and the oral mucosa

13

EMQs by Maju Mathews

Answers: Management of Skin Condition


33. (E)
Scabies presents with severe pruritus, which is more in the night, and
characteristic burrows can be seen on close examination.

34. (G)
Purple, plane polygonal, pruritic lesions on the ankle are characteristic of
Lichen planus.

35. (H)
The bullae in pemphigoid are subepidermal and are tense as compared to
those in pemphigus, which occur within the epidermis and are flaccid.

36. (C)
Acne Rosacea
Presence of pustules is an indication of treatment with antibiotics.

37. (H)
Pemphigus is often associated with oral lesions unlike pemphigoid.

14

EMQs by Maju Mathews

Theme: Antepartum Hemorrhage


Options
A.
B.
C.
D.
E.
F.
G.
H.
I.
J.
K.
L.
M.
N.
O.
P.

Endometriosis
Battledore placenta
Circumvallate placenta
Hydatidiform mole
Ectopic pregnancy
Pre eclampsia
Velamentous placenta
Placenta previa
Abruptio placentae
Septic abortion
UTI
Threatened abortion
Red degeneration
Uterine rupture
Inevitable abortion
Missed abortion

For each question below, choose the SINGLE most likely answer from the above list of
options. Each option may be used once, more than once, or not at all.

38. An 18-year-old girl at 11-weeks of gestation presents with bleeding PV


followed by abdominal pain. PV examination reveals closed cervical os.
39. A 40-year-old female presents with bleeding PV in 15 th week of gestation.
She has been having severe morning sickness. She has also noticed clots
and has passed some tissue per vaginam. She also complains of abdominal
pain. Abdominal examination reveals a fundal height corresponding to 20
weeks.
40. A 28-year-old female, 33-weeks pregnant, has back pain and severe lower
abdominal pain. FHS shows signs of distress.
41. A 32-year-old female, 32-weeks gestation is brought to A&E in shock. Shes
bleeding PV. The Foetal lie and presentation are abnormal. The FHS
appears to be normal at the moment. Likely diagnosis.
42. Vasa previa is likely to occur in __ type of placenta.

15

EMQs by Maju Mathews

Answers: Antepartum Hemorrhage


38. (L)
Rest is advised. Associated with subsequent preterm rupture of
membranes and preterm delivery.

39. (D)
Proliferated chorionic villi. Ultrasound shows snowstorm in the uterus.

40. (I)
Part of placenta become detached form the uterus. May cause Foetal
anoxia or death.

41. (H)
Placenta lies in the lower uterine segment and bleeding is always
revealed

42. (G)
Umbilical vessels pass within the membranes before insertion. If vessels
break Foetal blood is lost.

16

EMQs by Maju Mathews

Theme: Causes of Secondary Hypertension


Options
A.
B.
C.
D.
E.
F.
G.
H.
I.
J.

Alcohol intake
Pregnancy associated hypertension
Cushings syndrome
Hyperparathyroidism
Congenital adrenal hyperplasia
Renal artery stenosis
Aortic incompetence
Phaeochromocytoma
Acromegaly
Conns syndrome

For each question below, choose the SINGLE most likely answer from the above list of
options. Each option may be used once, more than once, or not at all.

43. A 65-year-old male smoker is noted to have a blood pressure of 160/90 and
suffers from peripheral vascular disease.
44. A 60-year-old female treated with bendrofluazide 2.5 mg/day for
hypertension has recently developed muscle cramps. The only abnormality
on investigations is a plasma potassium concentration of 3.0 mmol/l.
45. A 30-year-old obese woman with a history of polycystic ovarian disease
presents with 3 month history of weight gain, amenorrhoea and thirst. She
has a blood pressure of 145/90 and fasting plasma glucose of 7.2 mmol/I.
Investigations reveal a low LH and FSH, with high oestrogen and prolactin
concentrations.
46. A 23-year-old female with Turners syndrome is found to have a blood
pressure of 160/102.
47. A 26-year-old female with an ejection systolic murmur in the 2 nd right
intercostal space is noted to have a blood pressure of 180/100.
48. A 40-year-old female presents with palpitations, sweating and tightness in
the chest and pins & needle sensation. Urine screening test during the
attack shows presence of glucose.

17

EMQs by Maju Mathews

Answers: Causes of Secondary Hypertension


41.

(F)
Diagnosis of peripheral vascular disease in hypertension should alert one
to the risk of renal artery stenosis.

44.

(K)
Hypokalemia seen
hyperaldosteronism).

commonly

in

Conns

45.

(B)
Inverts tigstion scults suggest pregnancy.

46.

(I)
Recognized association of Turnners Syndrome.

47.

(I)
By exclusion of other diagnoses.

48.

(H)
Typical presentation of phaeochromocytoma

18

Syndrome

(primary

EMQs by Maju Mathews

Theme: Investigations
Options
A.
B.
C.
D.
E.
F.
G.
H.
I.
J.
K.
L.
M.
N.

X-ray hands
CT Brain
Skeletal survey
Bleeding time
Electromyogram & Nerve conduction studies
Arteriogram
Bone scan
MRI
Serum Ceruloplasmin Levels
Serum icon levels
PET
SPECT
Radioimmunoassay
No investigation required

For each question below, choose the SINGLE most likely answer from the above list of
options. Each option may be used once, more than once, or not at all.

49. A 35-year-old male typist has noticed that he is waking up during the night
with numbness and tingling in his fingers.
50. A 28-year-old male suffered a posterior knee dislocation following an
accident, which has been reduced successfully. Post reduction X-rays are
normal and there is no fracture. Which additional study should be ordered?
51. A 20-year-old is brought with tremors and facial dystonia. His father died of
cirrhosis of the liver. O/E he has brownish green discoloration around the
cornea.
52. A 4-year-old boy is brought with history of fall 5 days ago. The mother did
not think it was anything serious though he had been in pain. He has
several areas of ecchymosis on both arms and legs. X-ray shows a
displaced fracture of the tibia and an old healing fracture of the leg.
53. A 78-year-old woman with history of ischaemic heart disease is brought to
A&E with left sided weakness and eye deviation to the right.

19

EMQs by Maju Mathews

Answers: Investigations
49. (E)
Carpal tunnel syndrome. Common in people whose wrists are exposed to
regular friction. This is the study of choice.

50. (F)
To rule out vascular lesions which can prove dangerous.

51. (I)
Wilsons Disease.

52. (C)
Non-accidental injury, Always suspect this in a presentation like the
above scenario.

53. (B)
Features of Cerebrovascular accident. CT-Brain indicated to confirm
diagnosis and for mapping of site of lesion.

20

EMQs by Maju Mathews

Theme: Investigations in childhood urinary tract infection


Options
A.
B.
C.
D.
E.
F.
G.
H.
I.
J.
K.
L.

Plain Abdominal X-ray


MSU culture
Suprapubic aspirate culture
Bag urine culture
Clean catch urine culture
Ultrasound
DTPA scan
DMSA scan
Intravenous urogram
Micturating cystourethrogram (MCU)
Cystoscopy
MCU + DTPA scan

For each question below, choose the SINGLE most likely answer from the above list of
options. Each option may be used once, more than once, or not at all.

54. A 7-month-old infant presents with fever, vomiting and irritability. You
suspect UTI. What investigation will you usually do to confirm UTI?
55. The above investigation confirms the presence of UTI. What investigation
will you next carry out to ascertain the cause?
56. In the same patient the above investigation does not suggest any
abnormality. What investigation will you order next to reach the diagnosis?
57. A 3-year-old girl presents with a diagnosis of UTI. She has a history of
recurrent UTIs. Repeat ultrasound done shows a grossly dilated system.
What should be ordered for next?
58. A 7-year-old boy presents with an episode of renal colic, which you suspect
may be due to a calculus. Which is the preferred investigation to be carried
out?

21

EMQs by Maju Mathews

Answers: Investigations in childhood Urinary Tract Infection


54. (E)
Bag urine culture has many false positives.

55. (F)
Ultrasound detects a dilated system and also severe vesicoureteric reflux.

56. (H)
DMSA scan is done next in infants with a normal USG, which detects
scarring and obstructive uropathy.

57. (J)
MCU to detect reflux.

58. (A)
Most renal calculi are radio-opaque.

22

EMQs by Maju Mathews

Theme: Diagnosis of Neck Swelling


Options
A.
B.
C.
D.
E.
F.
G.
H.
I.
J.
K.
L.
M.
N.

Branchial cyst
Cystic hygroma
Sialoadenitis
Pleomorphic adenoma
Thyroglossal cyst
Simple goitre
Hodgkins Lymphoma
Ranula
Carotid body tumour
Medullary carcinoma of thyroid
Pharyngeal pouch
Cold abscess
Submandibular calculi
Salivary calculus

For each question below, choose the SINGLE most likely answer from the above list of
options. Each option may be used once, more than once, or not at all.

59. A 45-year-old man presents with a painless slow growing swelling at the
angle of his jaw. O/E a firm, non-tender, mobile swelling is seen.
60. A 59-year-old female presents with recurrent painful swelling of the angle of
the jaw present only during meals.
61. A 19-year-old female presents with a swelling protruding from the anterior
border of the upper third of the sternocleidomastoid. The swelling is
fluctuant but not trans-illuminable. She is otherwise well.
62. A 3-month-old baby has developed a trans-illuminant swelling occupying
the lower third of the neck. It enlarges when the child cries.
63. A 2-year-old child presents with swelling of the neck in the region of the
thyroid cartilage. The swelling moves upwards on protrusion of the
tongue.

23

EMQs by Maju Mathews

Answers: Diagnosis of Neck Swelling


59. (D)
Treat with surgery. A margin of normal gland is removed as well.

60. (M)
Submandibular calculi are the most common. Can be demonstrated by
plain radiography. Operative removal is the treatment of choice.

61. (A)
Arise form the vestigial remnants of the second branchial cleft. Treat with
excision.

62. (B)
Excision is the treatment of choice.

63. (E)
Excision is the treatment of choice. If left untreated, there is the risk of
infection.

24

EMQs by Maju Mathews

Theme Red eye


Options
A.
B.
C.
D.
E.
F.
G.
H.
I.
J.
K.
L.

Orbital Tumour
Glaucoma
Corneal Ulceration
Retinitis pigmentosa
Subconjunctival hemorrhage
Diabetic retinopathy
Orbital cellulitis
Episcleritis
Conjunctivitis
FB
Uveitis
Scleritis

For each question below, choose the SINGLE most likely answer from the above list of
options. Each option may be used once, more than once, or not at all.

64. This 36-year-old man on DVT prophylaxis was alarmed to see a red
uniform patch in his left eye. It was completely painless.
65. A 72-year-old man developed reduced vision, pain and circumcorneal
congestion on his right eye. O/E: hazy cornea and a fixed dilated pupil.
66. A 32-year-old lady developed sudden onset severe right-eye-pain,
photophobia, blurred vision, circumcorneal redness and lacrimation. O/E:
pain increased as the patient converged the eye and a small pupil.
67. A 67-year-old man a k/c of rheumatoid arthritis develops redness in the left
eye with a dull ache. O/E: Engorged vessels seen which can be easily
moved over the area.
68. A 21-year-old lady presents with itchy red eye, lacrimation and purulent
discharge involving both the eyes. O/E: hyperemic vessels seen which
can be easily moved over the area.

25

EMQs by Maju Mathews

Answers: Red eye


64. (E)
It is seen in-patient on anticoagulants and is a common accompaniment
of direct trauma to the eye.

65. (B)
Is associated with severe unilateral headache and vomiting.

66. (K)
It is associated with various connective tissue diseases.

67. (H)
Scleritis can cause thinning of sclera and is in danger of perforation on
severe cases scleromalacia perforans

68. (I)
It has to be treated with antibiotic drops and ointment to be applied at
night.

26

EMQs by Maju Mathews

Theme: Gynecology - Obstetrics Problems


Options
A.
B.
C.
D.
E.
F.
G.
H.
I.
J.
K.
L.
M.

Inevitable abortion
Hydatidiform mole
Ectopic pregnancy
Uterine rupture
Placenta previa
Abruptio placentae
Septic abortion
UTI
Threatened abortion
Red degeneration
Pre eclampsia
Endometriosis
Missed abortion

For each question below, choose the SINGLE most likely answer from the above list of
options. Each option may be used once, more than once, or not at all.

69. A 22-year-old presents with abdominal pain. Her LMP was 2 months back
and urine test for pregnancy was positive. In A&E, she developed mild
vaginal bleeding, and pain on passing water. Examination shows a tender
abdomen with cervical excitation.
70. A 42-year-old female, 28 weeks gestation presents with abdominal pain
with vomiting and low grade fever. Prior to conception, she had heavy &
prolonged periods with increase in frequency of urine.
71. A 23-year-old multipara has been in labour for twelve hours. She has mild
abdominal pain and vaginal bleeding. Her condition rapidly deteriorates &
she is in chock. On vaginal examination, the presenting part cannot be
felt. There are signs of Foetal distress.
72. A 24-year-old at 14 weeks of gestation presents with bleeding PV and
abdominal pain. On examination, the cervical os is dilated. She has
passed few clots and pieces of tissue.
73. An 18-year-old had and abortion at 14 weeks in a clinic a week back. Today
she developed severe abdominal pain, fever and discharge PV. On
examination she has a tender abdomen with guarding, rigidity, and
rebound tenderness.

27

EMQs by Maju Mathews

Answers: Gynecology obstetrics Problems


69. (C)
Fertilized ovum implants outside the uterine cavity. Always consider in a
woman with abdominal pain preceding bleeding PV. Emergency
management indicated.

70. (J)
Caused by thrombosis of a fibroids blood supply. Occurs in pregnancy.

71. (D)
Commonly due to dehiscence of caesarean section scar. Lower segment
less likely to rupture than classical scars. Continuos PPH with a well
contracted uterus is strongly suggestive.

72. (A)
Open os with severe symptoms.

73. (G)
Generally follows back-street abortion. Presents like acute salpingitis.

28

EMQs by Maju Mathews

Theme: Adverse effects of antihypertensives


Options
A.
B.
C.
D.
E.
F.
G.
H.
I.
J.
K.

Sodium Nitroprusside
Hydralazine
Prazosin
Minoxidil
Verapamil
Bendrofluazide
Clonidine
Enalapril
Alpha methyldopa
Nifedipine
Atenolol

For each question below, choose the SINGLE most likely answer form the above list of
options. Each option may be used once, more than once, or not at all.

74. A 35-year-old lady following treatment for eclampsia developed a


photosensitive rash over the face with joint pains and fever.
75. A 50-year-old male being treated for essential hypertension develops
sudden severed pain at the great toe. On examination joint is swollen
tender and red-hot.
76. A 60-year-old lady developed dry cough following administration of this
antihypertensive.
77. A 43-year-old man presents to A&E following a transient ischaemic attack
and BP recorded is 180/105-mmHg. He was on anti-hypertensive
medication since 6 months with regular follow-ups. He has not taken his
medication since 2 days as he was on holiday.
78. Heart block if co-administrated with beta-blockers.

29

EMQs by Maju Mathews

Answers: Adverse Effects of Antihypertensives


74. (B)
Can cause drug induced SLE. Anti nuclear factor is negative.

75. (F)
Diuretics can precipitate gout.

76. (H)
This is a common side effect of ACE inhibitor therapy.

77. (G)
Clonidine is a centrally acting antihypertensive and can cause rebound
hypertension.

78. (E)
Co administration of verapamil and beta-blockers must be avoided.

30

EMQs by Maju Mathews

Theme: Nutritional deficiencies


Options
A.
B.
C.
D.
E.
F.
G.
H.
I.
J.
K.

Ascorbic acid
Niacin
Copper
Manganese
Vitamin-A
Vitamin-K
Zinc
Thiamine
Vitamin B12
Vitamin-D
Chromium

For each question below, choose the SINGLE most likely answer form the above list of
options. Each option may be used once, more than once, or not at all.

79. An 85-year-old woman living alone is lethargic and weak. She also has
bleeding gums and joint pains.
80. A 78-year-old woman on Total-parental-nutrition has a rash and
eczematous scaling of the perineum and face.
81. A 45-year-old chronic alcoholic is confused and has nystagmus, ataxia, and
peripheral neuropathy.
82. A patient on total parental nutrition develops pancytopenia.
83. A 78-year-old woman on high dose antibiotics is deficient in a vitamin
synthesized by intestinal flora.

31

EMQs by Maju Mathews

Answers: Nutritional Deficiencies


79. (A)
Scurvy; deficiency of Vitamin-C.

80. (G)
Zinc deficiency, common in people on TPN without supplementation.

81. (H)
Thiamine deficiency is common in chronic alcohol consumption and leads
to Wernickes encephalopathy and Korsakoffs psychosis.

82. (C)
Deficiency of copper can occur in patients on TPN.

83. (F)
Vitamin K is synthesized by the intestinal flora and is necessary for
coagulation of blood.

32

EMQs by Maju Mathews

Theme: Causes of haematuria


Options
A.
B.
C.
D.
E.
F.
G.
H.
I.
J.
K.
L.

Cystitis
Renal tract calculi
Renal cell carcinoma
Bladder carcinoma
Good pastures syndrome
Wegeners granulomatosis
Prostatitis
Post retention
Infective endocarditis
Glomerulonephritis
Interstitial nephritis
Polycystic kidney disease

For each question below, choose the SINGLE most likely answer from the above list of
options. Each option may be used once, more than once, or not at all.

84. A 50-year-old female develops haematuria with exquisite right loin pain. On
examination abdomen is soft with tenderness over the right lumber region.
She is afebrile.
85. A 40-year-old man with hypertension presents with frank haematuria.
Physical examination shows a palpable mass over right lumber area.
Other biochemical abnormalities are: mildly elevated urea and creatinine.
He gives a history that his uncle recently underwent a renal transplant the
cause for which he is unaware of and his mother died of renal failure.
86 A 65-year-old male presents with haematuria, right loin pain, and night
sweats. Physical examination reveals a mass in the right loin. Preliminary
investigations slow a normochromic normocytic anaemia.
87. A 15-year-old boy with recurrent episodes of haematuria following a flu-like
illness. He is otherwise well and examination is normal.
88. A 30-year-old man comes with frank haematuria and haemoptysis. Chest Xray shows bilateral infiltrates in the lower zones.
89. A 42-year-old man presents with haematuria frequency dysuria lower
abdominal pain and fever. Abdomen is non-tender but rectal examination
is very painful.

33

EMQs by Maju Mathews

Answers: Causes of haematuria


84. (B)
Stones in the kidney will cause loin pain. In the ureter cause colic, bladder
cause strangury, urethra cause obstruction.

85. (L)
Adult polycystic kidney is a autosomal dominant condition. There is a
gradual decline in renal function and is common cause of CRF.

86. (C)
Classical triad of loin pain, abdominal mass and haematuria.

87. (J)
IgA disease is a common cause of recurrent haematuria in young men
and it often follows viral infection.

88. (E)
Proliferative glomerulonephritis and lung symptoms caused by antibasement membranae antibodies.

89. (G)
Non gonococcal urethritis is a commonest case of genitourinary infection
in males.

34

EMQs by Maju Mathews

Theme: Bleeding PV
Options
A.
B.
C.
D.
E.
F.
G.
H.
I.
J.
K.
L.

Adenomyosis
Anaemia
DUB
Endometrial carcinoma
Endometriosis
Chronic Pelvic infection
Bartholins abscess
Pregnancy
IUCD
Cancer Cervix
Fibroids
Hypothyroidism

For each question below, choose the SINGLE most likely answer from the above list of
options. Each may be used once, more than once, or not at all.

90. A middle-aged female with excessive periods since the last 1-year c/o
weight gain, edema and constipation. She has poor appetite and is infertile.
91. A 56-year-old postmenopausal obese nulliparous female with heavy
bleeding since the last four weeks is most likely to have.
92. A couple has been trying for a child for 4 years. The female is 32-years-old
has dysmenorrhea and dyspareunia. PV examination reveals a fixed
retroverted uterus with nodules in the uterosacral ligaments. The most likely
cause for the symptoms.
93. A 15-year-old girl with menorrhagia but no dyspareunia, discharge.
Abdominopelvic USG is normal. No other obvious pathology is apparent.
94. A 55-year-old female perimenopausal with IDDM, mother had colonic
carcinoma, now uses about 35-40 pads during her periods, which lasts for
more than a week.
95. A 42-year-old female on combined OCPs with heavy & prolonged periods,
with increase in frequency of urine. She has been given mefenamic acid,
progesterones, and danazol but to no avail. The cause for her troubles is
likely to be.
96. This couple has been trying for 4 years. The female 32 years old, has
dysmenorrhea and dyspareunia. PV (per vaginam) examination reveals an
enlarged boggy tender uterus.

35

EMQs by Maju Mathews

Answers: Bleeding PV
90. (L)
Typical features of hypothyroidism.

91. (D)
Classical presentation

92. (E)
Foci of endometrial glandular tissue found beyond the uterine cavity. If
asymptomatic, no treatment required. Otherwise treat with hormones
(Danazol, Buserelin) or surgery.

93. (C)
Bleeding without known cause.

94. (D)
Always consider in postmenopausal females.

95. (K)
Benign tumors of smooth muscle present with menorrhagia, infertility,
mass or pain.

96. (A)
Endometrial tissue within the uterine muscle wall.

36

EMQs by Maju Mathews

Theme: Gynecological Malignancies


Options
A.
B.
C.
D.
E.

Ovarian carcinoma
Vulval carcinoma
Cervical carcinoma
Endometrial carcinoma
Vaginal carcinoma

For each question below, choose the SINGLE most likely answer form the above list of
options. Each option may be used once, more than once, or not at all.

97. Presents with pruritus.


98. Presents with ascites.
99. Presents with postmenopausal bleeding.
100. Presents with post coital bleeding.
101. Causes most deaths.

37

EMQs by Maju Mathews

Answers: Gynecological Malignancies


97. (B)
Common mode of presentation.

98. (A)
Or mass abdomen with associated general signs of malignancy.

99. (D)
Always think of CA endometrium in postmenopausal bleeding.

100. (C)
Patient screening with regular cervical smear is important.

101. (A)
Though this is not the most common gynecological malignancy. It causes the
most deaths because it is frequently asymptomatic in the early stage and
hence results in late diagnosis.

38

EMQs by Maju Mathews

Theme: Management of hypertension


Options
A.
B.
C.
D.
E.
F.
G.
H.
I.
J.
K.

Hydralazine
Calcium antagonist
Sodium nitroprusside
Diuretic (thiazide)
Alpha-blocker
Alpha-methyldopa
Angiotensin-2 receptor blocker
Beta blocker
ACE inhibitor
Minoxidil
None

For each question below, choose the SINGLE most likely answer form the above list
of options. Each option may be used once, more than once, or not at all.

102. A 38-year-old pregnant female is noted to have blood pressure of 145/90


at 30 weeks gestation. Urine protein is negative and she is generally
asymptomatic.
103. A 60-year-old male is newly diagnosed with type-2 diabetes with fasting
plasma glucose of 7.6 mmol/l, HbA1c of 6.8% negative urinalysis and
blood pressure of 130/80.
104. A 70-year-old following a transient ischaemic attack is noted to have an
average blood pressure of 160/80. His past history includes dyslipidemia
(for which he takes a fibrate) and gout.
105. A 68-year-old patient is noted to be consistently hypertensive with a mean
blood pressure of 165/100. In his past history he was diagnosed with
asthma but does not take any therapy.
106. A 30-year-old male type-1 diabetic on annual assessment is noted to have
a blood pressure of 120/80 but a urine protein excretion of 0.4-g/day.

39

EMQs by Maju Mathews

Answers: Management of Hypertension


102. (F)
Methyldopa is the first line treatment for pregnancy-induced hypertension.

103. (K)
No indication for antihypertensive therapy

104. (B)
Diuretics are contraindicated in gout.

105. (D)
Since beta-blockers are contraindicated, a low course diuretic should be
tried.

106. (I)
Though the blood pressure is normal the patient has diabetic nephropathy
indicated by he microalbuminuria and hence should be treated with an
ACE inhibitor.

40

EMQs by Maju Mathews

Theme: Contraceptives of choice


Options
A.
B.
C.
D.
E.
F.
G.
H.
I.

Condom
Depot hormonal contraception
Diaphragm
Spermicides
IUCD
Natural method
Implant hormonal contraception
COCP
No contraception required

For each question below, choose the SINGLE most likely answer form the above list of
options. Each option may be used once, more than once, or not at all.

107. In a female of 24 years of age in a stable relationship seeking


contraception for a few years, with one child and a history of migraine.
108. For a female of 20 with a previous history of ectopic pregnancy, no
menstrual problems, any other medical conditions or family history of
cancer.
109. For a female with multiple partners, many short term relationships.
110. In a single female in stable relationship seeking a no fuss contraceptive
method lasting for four to five years but easily reversible. She does not
have any menstrual problems.
111. A 30-year-old female has just undergone sterilization. She is worried about
immediate contraceptive cover.

41

EMQs by Maju Mathews

Answers: Contraceptives of Choice


107. (E)
Oral contraceptives can exacerbate migraine.

108. (H)
IUCDs predispose to ectopics.

109. (A)
Prevents sexually transmitted diseases.

110. (G)
5 years contraception with one implantation

111. (A)
An IUCD should be left in place till the next period, if this is not done than
barrier is the most effective and easy choice.

42

EMQs by Maju Mathews

Theme: Causes of chronic renal failure


Options
A.
B.
C.
D.
E.
F.
G.
H.
I.
J.
K.
L.

Diabetic nephropathy
Hypertensive nephropathy
Bladder outflow obstruction
Glomerulonephritis
Gout amyloidosis
Amyloidosis
Chronic pyelonephritis
Interstitial nephritis
Hypercalcemia
Systemic vasculitis
Nephrotoxic drugs
Polycystic kidney disease

For each question below, choose the SINGLE most likely answer form the above list of
options. Each option may be used once, more than once, or not at all.

112. A 50-year-old man presents with an episode of renal colic. He also gives a
history of intermittent knee pain.
113. A 55-year-old woman is found to have chronic renal impairment on a
routine blood test prior to a coronary angiogram. She is on no medication
apart from anti-anginals.
114. A 40-year-old woman being investigated for hypertension is found on
routine blood test to have chronic renal impairment. She suffered recurrent
UTIs, as a child but has been asymptomatic since.
115. A 40-year-old woman has renal failure and proteinuria. She also complains
of visual impairment and numbness over her peripheries.
116. A 50-year-old man suffers daily headaches subsequent to an accident at
work sustained four years previously. He now complains of increasing
malaise and joint pains and is found to have renal impairment.
117. A 65-year-old man presents with a two-year history of urinary frequency,
hesitancy and poor stream. On examination he has a palpable mass
arising from the pelvis extending half way to the umbilicus and dull on
percussion.

43

EMQs by Maju Mathews

Answers: Causes of Chronic Renal Failure


112. (E)
Diagnosis is by raised serum uric acid and presence of negatively
birefringently crystals in polarized light in the synovial fluid.

113. (B)
Seen in long standing poorly controlled patients.

114. (G)
Present as CRF or one of its complications and probably arises form
children UTI associated with reflux and scarring.

115. (A)
Main feature is proteinuria starts as intermittent microalbuminuria, which
leads to constant proteinuria and occasionally nephrotic syndrome.

116. (K)
Prolonged use of analgesics can cause analgesic nephropathy.

117. (C)
USG or CT scan may be required to ascertain the cause of the mass.

44

EMQs by Maju Mathews

Theme: Management of psychiatric illness


Options
A.
B.
C.
D.
E.
F.
G.
H.
I.

Psychodynamic Psychotherapy
Family Therapy
Cognitive Therapy
Exposure and Response prevention (Behavioral Therapy)
Systematic Desensitizations (Behavioral therapy)
Electro convulsive Therapy
Social skills training
Relaxation therapy
Anxiety Management

For each question below, choose the SINGLE most likely answer form the above list of
options. Each option may be used once, more than once, or not at all.

118. A 33-year-old lady presents with a long history of depression. She works
as a shop assistant and has a good appetite and sleeps 8 hours per night.
She has never had suicidal thoughts.
119. A couple brings their 12-year-old son to the clinic as he is having problems
at school and is always getting into trouble. While describing his problems,
the parents start arguing with other and blame each other for the boys
problems.
120. A 30-year-old lady presents with dermatitis of both her hands. She is very
particular about cleanliness and has to wash her at least 20 times per day.
121. A 38-year-old man presents with low mood, which started after he had an
accident 10 months ago. Ha has frequent nightmares and flashbacks of the
incident and is constantly thinking about it.
122. A 35-year-old lady who refuses to go out, as she is phobic of dogs.

45

EMQs by Maju Mathews

Answers: Management of Psychiatric illness


118. (C)
This is NOT a case of clinical depression. Just having low mood does not
meet the criteria for depression. This is case of dysthymia. The
intervention of choice would be cognitive therapy. This form of therapy
encourages people to look at the maladaptive patterns of thinking that
makes them depressed. It is carried out by a therapist over 16 to 20
sessions and the patient has to keep a diary and do home works etc. it is
used for depression, anxiety, dysthymia, PTSD etc.

119. (B)
Many of the problems especially that children have result dysfunctional
family interactions. In these situations it is family therapy that would be
the therapy of choice.

120. (D)
This lady has OCD. The therapy would involve encouraging her to make
herself dirty (exposure) and letting her remain dirty and not allowing her
to wash herself (response prevention). This has to be repeated over a
number of sessions. Behavior is being altered and hence it is a form of
Behavior therapy.

121. (C)
PTSD. See explanation to 1.

122. (E)
Getting rid of a phobia by gradual exposure to the feared stimulus, along
with relaxation.

46

EMQs by Maju Mathews

Theme: Mechanism of action of antihypertensives


Options
A.
B.
C.
D.
E.
F.
G.
H.
I.
J.

Alpha methyldopa
Bendrofluazide
Carvedilol
Prazosin
Hydralazine
Losartan
Minoxidine
Nebivolol
Nifedipine
Quinapril

For each question below, choose the SINGLE most likely answer form the above list
of options. Each option may be used once, more than once, or not at all.

123. A combined alpha and beta-receptor antagonist


124. A selective beta-1 receptor antagonist
125. Blocks the angiotensin-II receptor
126. Acts by reducing central sympathetic nervous system activity
127. A specific alpha- receptor antagonist

47

EMQs by Maju Mathews

Answers: Mechanism of action of antihypertensives


123. (C)
This has both alpha and beta blocking effects.

124. (I)
This is a selective beta-1-antagonist and hence has reduced beta-2
mediated effects like cold peripheries.

125. (F)
Losartan, Eprosartan and irbesartan all of these are angiotensin
antagonists.

126. (H)
This is an imidazoline agonist acting on the sympathetic nuclei in the
medulla reducing sympathetic activity.

127. (D)
Doxazosin is also an alpha-1 antagonist.

48

EMQs by Maju Mathews

Theme: Management of antepartum hemorrhage


Options
A.
B.
C.
D.
E.
F.
G.
H.
I.

Platelets
FDP
Caseation section
Basophils
Set up ivi with rapid saline infusion
Amniotomy to deliver the child
Oxytocics
Tocolytics
Prostaglandins

For each question below, choose the SINGLE most likely answer form the above list of
options. Each option may be used once, more than once, or not at all.

128. A 33-year-old, 34 weeks gestation is brought in shock. The first step of


management would be______
129. A 32-year-old, 32 weeks gestation is brought to A&E in shock. She is
bleeding PV. Her blood pressure the previous evening was 150/100 when
measured at home. But the shock appears to be out of proportion to the
bleeding. She is started on a saline infusion your next step would be.
130. In a case of Abruptio placenta, is recommended in the third stage of
labour.
131. A 24-year-old primi presents with, rupture of membranes without onset of
labour. The most appropriate method of induction would be.

49

EMQs by Maju Mathews

Answers: Management of Antepartum Hemorrhage


128. (E)
Adequate resuscitation of patient most important.

129. (B)
Given to normalize APTT and PT.

130. (G)
To reduce bleeding.

131. (G)
To reduce the possibility of ascending infection.

50

EMQs by Maju Mathews

Theme: Normal childhood development


Options
A.
B.
C.
D.
E.
F.
G.
H.
I.
J.

One month
Three months
Six months
Nine months
One year
Two Years
Three years
Four Years
Six years
18 months

For each question below, choose the SINGLE most likely answer form the above list of
options. Each option may be used once, more than once, or not at all.

132. Stands unsupported.


133. Transfers objects from one hand to the other.
134. Smiles spontaneously.
135. Builds tower of 2 cubes.
136. Dresses without supervision.
137. Says dada or mama appropriately.

51

EMQs by Maju Mathews

Answers: Normal Childhood Development


132. (E)
Neat pincer grip and able to speak dada or mama.

133. (C)
Stands holding on and pulls to sit without head lag.

134. (B)
Prone with head up to 90 degree and able to bring hands together

135. (J)
75% of children able to build 2 towers and able to walk backwards.

136. (H)
Catches bouncing ball.

137. (E)
Neat pincer grip and able to speak dada or mama.

52

EMQs by Maju Mathews

Theme: Side effects of psychotropic


Options
A.
B.
C.
D.
E.
F.
G.

Monoamine oxidase inhibitors


Lithium
Sodium Valproate
Carbamazepine
Haloperidol
Selective serotonin reuptake inhibitor
Tricyclic antidepressants

For each question below, choose the SINGLE most likely answer form the above list of
options. Each option may be used once, more than once, or not at all.

138. A 42-year-old male was started medication for depression. One morning
he collapses as soon as he gets up from bed.
139. A 32-year lady with depression has nausea, vomiting and GI upset a few
hours after taking her medication.
140. A 24-year-old male is started on medication. Three days later he develops
fever, rigidity, and has a fluctuating blood pressure. He becomes confused
and his CPK is raised.
141. A 28-year-old male develops severed spasm of his neck muscles soon
after he takes his medication.
142. A 34-year-old patient with depression on medication develops a headache
sometime after eating a sandwich. His blood pressure is noted to be
220/150-mmHg.

53

EMQs by Maju Mathews

Answers: Side Effects of Psychotropic


138. (G)
Postural hypotension is common side effect of Tricyclic antidepressants.

139. (F)
GI upset is a common side effect of SSRIs (Fluoxetine). Other side
effects are insomnia and an increase in anxiety.

140. (E)
Neuroleptic malignant syndrome is a side effect of neuroleptics with the
above features. Other side effects of neuroleptics are acute dystonias,
tremors, parkinsonian features & tardive dyskinesia.

141. (E)
Acute dystonia

142. (A)
Cheese reaction with MAOI. Foods containing tyramine should be
avoided by patients taking MAOI, eg. cheese, red wine, Avocado etc.

54

EMQs by Maju Mathews

Theme: Dementia
Options
A.
B.
C.
D.
E.
F.
G.
H.
I.
J.
K.
L.

Picks Diseases
Parkinsons Disease
Huntingtons Disease
Subdural haematoma
HIV
Alcoholic Dementia
Vascular Dementia (Multi infarct dementia)
Dementia of frontal lobe type
Lewy body Dementia
Creutzfeldt-Jacob Disease
Alzheimers Disease
Normal Pressure hydrocephalus

For each question below, choose the SINGLE most likely answer form the above list of
options. Each option may be used once, more than once, or not at all.

143. A 68-year-old lady has been brought to the clinic. Her daughter says that
over the past 3 years, she has become increasingly forgetful and they
have to repeat things, which they have just told her. Over the past few
months she has also not been able to dress herself not eat with a fork and
knife. Overall there has been a gradual decline in her level of functioning.
144. A 42-year-old bank clerk is sent from his work place for an assessment as
he has become inefficient at work and is becoming forgetful, in the clinic,
he is uncomfortable and has twitchings in his face. His hand movements
are also jerky. His father died at the age of 46 of unknown cause.
145. A 36-year-old man became forgetful and disorientated about 2 years ago.
His mental state showed marked fluctuation with disorientation and
bizarre memory disturbances alternating with periods of lucidity. He was
emotionally revealed myoclonic jerks, apraxia & choreoathetoid
movements. He was also incontinent of urine and feces.
146. A 49-year-old man presents with lethargy, easy fatigability, and vague
weakness in the legs. For past 6 months, his family has noted him to be
dull & forgetful. He was slow in motor responses with mild impairment of
memory. Of late, he has also become incontinent of urine.
147. A 72-year-old man had sudden weakness in his right upper limb and could
not move it for 8 hours. It recovered on its own. His memory has been
poor since then. A week ago, he had another similar episode and his
memory has deteriorated still further. He is also being treated for
hypertension.

55

EMQs by Maju Mathews

Answers: Dementia
143. (K)
Most common cause of dementia.

144. (C)
Autosomal dominant. Presents with chorea, tremors, personality changes
psychiatric disorders, rigidity & fits in any order.

145. (J)
Equal in males & females. Features are personality changes fatigue,
slowness depression, memory impairment, psychosis, seizures,
myoclonic, jerks, cortical blindness, not in any particular order.

146. (L)
Memory impairment, slowness, apathy, unsteady gait and incontinence
are features of this condition. Treatment is ventriculoatrial shunt to lower
intraventricular pressure.

147. (G)
Multiple minor strokes can result in dementia.

56

EMQs by Maju Mathews

Theme: Substance Misuse


Options
A.
B.
C.
D.
E.
F.
G.
H.

Delirium Tremens
Opiate withdrawal
Benzodiazepine withdrawal
Stimulant (Cocaine, Amphetamine, Ecstasy) Abuse
Opiate dependence
Alcohol dependence
LSD
Harmful alcohol abuse

For each question below, choose the SINGLE most likely answer form the above list of
options. Each option may be used once, more than once, or not at all.

148. A 28-year-old man who lives alone, presents to A&E with pain abdomen.
He is vomiting and sweating profusely. He also has diarrhea. He has not
been eating for the past 3 days, as he did not have any money. His pupils
are dilated.
149. A 48-year-old man was admitted to the medical ward three days ago.
Today he is shaky, sweaty, and scared and says that the devil is here to
get me for my sins and is picking at imaginary objects.
150. An 18-year-old girl is brought to A&E at 6-am. In the morning after she
collapsed in a nightclub. She spent the whole night dancing.
151. A 45-year-old male is brought to the clinic by his wife as their marriage is
on the rocks because of his heavy drinking. He admits to drinking 2 bottles
of whiskey everyday for the past 6 years. He gets shakes when he wakes
up and has to drink to steady himself and continues to drink through the
day.
152. Multiple injection marks and pinpoint pupils are features of

57

EMQs by Maju Mathews

Answers: Substance Misuse


148. (B)
These are the typical features of opiate withdrawal. Not having money buy
the substances and dilated pupils suggest the diagnosis.

149. (A)
In all patients, irrespective of the condition they are being treated for it is
important to take an alcohol history.

150. (D)
Ecstasy use is common among night-clubbers. It can cause cardiac
arrhythmias and death from dehydration.

151. (F)
Features of Alcohol dependence syndrome are (Any 3 for diagnosis).
Stereotyped pattern of drinking
Prominence of drink seeking behavior
Increased tolerance to alcohol
Withdrawal symptoms
Compulsion to drink
Relief of withdrawal symptoms
Reinstatement after abstinence

152. (E)
Opiate dependence

58

EMQs by Maju Mathews

Theme: Treatment of stroke


Options
A.
B.
C.
D.
E.
F.
G.
H.
I.
J.
K.

Percutaneous endoscopic gastronomy (PEG) feeding


Physiotherapy
Observe and monitor patient
Nasogastric tube (NG) feeding
Lipid lowering agent
Referral to psychiatrist
Warfarin
Aspirin and dipyridamole
Graduated compression stocking
Anti-hypertensive agent
Vascular or neuro-surgery

For each question below, choose the SINGLE most likely answer form the above list of
options. Each option may be used once, more than once, or not at all.

153. A 76-year-old woman presents with a two-week history of right sided


weakens which has more or less resolved. Computed tomography of head
confirms cerebral infraction. Her pulse is irregularly irregular, but her heart
sounds are normal. She is not on any medication.
154. A 76-year-old woman is found collapsed at home. On arrival at hospital
her Glasgow, coma scale score is 3. Her computed tomography of head
shows cerebral hemorrhage.
155. A 56-year-old man has been waking up in the early hours of the morning
and at times, been tearful since his stroke 2 months ago. However he
denies depression. On examination he is apathetic and has a dense right
hemiparesis.
156. A 56-year-old man presents with a transient slurring of speech and
dropping of the mouth. He is normally fit and well. A carotid ultrasound
Doppler scan has confirmed 80% stenosis of the origin of right internal
carotid artery. All his other investigations have been normal.
157. A 67-year-old man has been admitted into hospital for the last 1 week with
a dense left hemiparesis. He has kept nil-by-mouth since his admission,
because of his unsafe sallow.

59

EMQs by Maju Mathews

Answer: Treatment of Stroke


153. (G)
Warfarin reduces the risk of primary and secondary strokes. It is more
effective than aspirin in patients with atrial fibrillation.

154. (C)
Anticoagulants are contraindicated and antiplatelet therapy should be
avoided. The patient is comatose and should managed conservatively.

155. (F)
Post stroke depression a common entity affecting 30- 40% of patients with
stroke.

156. (K)
Endarterectomy is indicated in symptomatic patients with extracranial
carotid stenosis of more than 70%.

157. (D)
Patients should start receiving feeds if their swallowing is impaired,
initially with NG feed and later with a PEG.

60

EMQs by Maju Mathews

Theme: Infertility treatment


Options
A.
B.
C.
D.
E.
F.
G.
H.
I.
J.
K.

Surgery
GIFT
20 %
IVF
LHRH analogues
AID (Artificial insemination Donor)
10 %
Bromocriptine
Clomiphene
AIH (Artificial insemination husband)
Gonadotrophins

For each question below, choose the SINGLE most likely answer form the above list of
options. Each option may be used once, more than once, or not at all.

158. The method of choice for assisted fertilization in tubal problems


159. Method of choice for assisted fertilization when tubes are patient
160. In an infertile with pituitary microadenoma, the treatment is ___
161. Initial treatment for anovulation would be with __
162. For a couple trying for a child where the male partner is diagnosed to
have impotence the best method to be offered would be __

61

EMQs by Maju Mathews

Answers: Infertility Treatment


158. (D)
Embryos are returned to the uterus, done as an outpatient procedure.

159. (B)
Gametes placed in the tubes by laparoscopic cannulation

160. (H)
Surgery generally not indicated

161. (I)
To stimulate follicle development

162. (J)
Artifical insemination by sperm

62

EMQs by Maju Mathews

Theme: Causes of hallucinations & delusions


Options
A.
B.
C.
D.
E.
F.
G.
H.
I.

Schizophrenia
Psychotic depression
Mania
Toxic confusional state
Delirium tremens
Korsakoffs psychosis
Paraphrenia
Drug-induced personality disorder
Hysteria

For each question below, choose the SINGLE most likely answer form the above list of
options. Each option may be used once, more than once, or not at all.

163. A 35-year-old man is agitated and euphoric. He claims to be helping the


Prime minister with economic policy, although this is not true when
checked.
164. A 20-year-old man complains that all his movements are being watched.
Sometimes he feels as though his actions are being controlled by his
radio. At other times he is aware of voices describing what he is doing.
165. A 50-year-old man complains of being pursued, by the police for a crime
he denies committing. He has poor concentration and impaired short-term
memory. He admits to drinking large amounts of alcohol.
166. A 65-year-old woman says that she died three months ago and is very
distressed that nobody has buried her. When she is outdoors, she hears
people say that she is evil and needs to be punished.
167. A 40-year-old tee-total woman is recovering from a hysterectomy two days
ago. At night she becomes agitated and complains of seeing animals and
children walking round the ward.

63

EMQs by Maju Mathews

Answers: Causes of hallucinations and delusions


163.(C)
Other features include increased appetite, disinhibition, increased sexual
drive, decreased insight and grandiose delusion.

164.(A)
First rank symptoms (seen in 70%), Thought insertion, thought
broadcasting, thought withdrawal, passivity feeling, primary delusions,
somatic hallucinations and hearing voices (3 rd person).

165. (F)
May follow Wernickes encephalopathy and is due to thiamine deficiency
and is associated to with confabulation and decreased ability to acquire
new memory.

166. (B)
This is cotords syndrome, depression of psychotic features, mainly, in
elderly.

167.(D)
Common in hospitalized patients.

64

EMQs by Maju Mathews

Theme: Syndromes
Options
A.
B.
C.
D.
E.
F.
G.
H.
I.
J.

Treacher-Collin Syndrome
Allports Syndrome
Edwards Syndrome
Downs Syndrome
Laurence Moon Biedl Syndrome
Noonans Syndrome
Turners Syndrome
Klinefelters Syndrome
Prader wills Syndrome
Williams Syndrome

For each question below, choose the SINGLE most likely answer form the above list of
options. Each option may be used once, more than once, or not at all.

168. A 13-year-old girl is short with wide carrying angle, apubertal. On


examination there is rediofemoral delay.
169. An 8-year-old child with ptosis, down slanting eyes, low set ears, webbed
neck and easy bruising. On examination there is a Pansystolic murmur.
170. A newborn baby low set malformed ears, protruding eyes & rockerbottom
feet.
171. A newborn baby who is hypotonic and very sleepy. On examination, the
baby has blue eyes and blond hair.
172. A newborn with lower eyelid notching oblique palpebral fissures, flat
malar bones and absent zygoma.

65

EMQs by Maju Mathews

Answers: Syndromes
168. (G)
Seen in girls, lack of a sex chromosome. Gonads are rudimentary or
absent.

169. (F)
Autosomal dominant, height and intelligence are below average.

170. (C)
Trisomy-18, rigidity, mental retardation, cleft lip, hernias, and short
sternum are other features. The index finger overlaps the third digit.

171. (I)
Lesion on chromosome-15. Later on in life, they develop hyperphagia,
obesity, hypogonadism, short stature and hyperglycemia.

172. (A)
These are the main feature.

66

EMQs by Maju Mathews

Theme: Treatment of movement disorders


Options
A.
B.
C.
D.
E.
F.
G.
H.
I.
J.
K.

Beta blockers
Tetrabenazine
Levodopa
Botulinum toxic
Physiotherapy
Apomorphine
Procyclidine
Amantadine
Baclofen
Penicillamine
Withdraw drug

For each question below, choose the SINGLE most likely answer form the above list of
options. Each option may be used once, more than once, or not at all.

173. A 30-year-old asthmatic is admitted with exacerbation of asthma, and has


made a good recovery with nebulized bronchodilators and oral steroids.
However she has noticed a fine tremor affecting her hands since
admission. The best way to manage the tremors would be.
174. A 25-year-old man presents with dysarthria, tremor, involuntary
movements and chronic liver disease. He is noted to have KayserFleischer rings affecting his eyes.
175. A 45-year-old man with known Huntingtons disease is distressed with his
choreiform movements. He has not been tried on any medication yet.
176. A 15-year-old girl was recently started on metoclopramide for nausea,
following a bout of gastroenteritis. Soon after taking her first tablet, she
had a severe reaction. Her eyes were forced upwards and she lost
voluntary movements of her eyes.
177. A 67-year-old man presents with one-year history of worsening resting
tremor. Both his muscle tone and walking is normal. He finds the tremor
embarrassing.

67

EMQs by Maju Mathews

Answers: Treatment of Movement Disorders


173. (K)
Salbutamol causes tremors. Treatment includes or stopping the drug.

174. (J)
Wilsons disease, Treat with penicillamine.

175. (B)
Used in controlling movement disorders in Huntingtons, Hemiballismus
and chorea.

176. (G)
Oculogyric crisis due to extrapyramidal effects of metoclopramide. Can be
treated with anticholinergics.

177. (A)
Begin essential tremor. Affects hand and trunk. Alcohol & beta-blockers
relieve symptoms.

68

EMQs by Maju Mathews

Theme: Management of pelvic inflammatory disease PID


Options
A.
B.
C.
D.
E.
F.
G.
H.

Monilial vaginitis
Trichomonas vaginalis
Foreign body
Atrophic vaginitis
Endometrial carcinoma
Normal discharge of pregnancy
Normal periods
Ovulation

For each question below, choose the SINGLE most likely answer form the above list of
options. Each option may be used once, more than once, or not at all.

178. An 8-year-old child is brought with offensive discharge per vaginam since
the past three months. She has received three courses of antibiotics from
the GP but to no avail. The external genitalia appear normal except for the
purulent discharge.
179. A 65-year-old postmenopausal female has had a profuse, purulent and
occasionally blood stained discharge per vaginam with some discomfort
and soreness at the vulva. The GP has treated her with ethynylestradiol
but the response has been poor. The condition to be ruled out is.
180. A 24-year-old female presents with intense vulval irritation and thick white
vaginal discharge.
181. A 20-year-old, 28 weeks gestation, presents with sudden onset of scanty
yellow discharge with mild vulval irritation. The vaginal wall has multiple
punctate ulcers.
182. A 58-year-old presents with profuse purulent discharge. The vaginal walls
are inflamed. Microscopy does not reveal any organism.

69

EMQs by Maju Mathews

Answers: Management of Pelvic Inflammatory Disease


178. (C)
Unlikely to be due to STD as the external genitalia appears normal. The
resistance to treatment and the age of the patient makes the diagnosis
possible.

179. (E)
Bleeding PV in a postmenopausal woman is CA endometrium until proven
otherwise. It is important to rule out this condition in bleeding or if
improvement of senile vaginitis is not achieved rapidly following treatment.

180. (A)
Characteristic discharge. Confirm diagnosis by microscopy.

181. (B)
Trichomoniasis may manifest during pregnancy. The characteristic
discharge is profuse, offensive, yellow and frothy.

182. (D)
In the absence of bleeding, the first diagnosis to be considered in this
case is atrophic vaginitis.

70

EMQs by Maju Mathews

Theme: Fitness to drive


Options
A.
B.
C.
D.
E.
F.
G.
H.
I.
J.
K.

Banned from driving


Should not drive for 1 month
Should not drive for 2 months
Must not drive for 3 months
Must no drive for one year
May not drive for two years
May not drive for three years
Should not drive for 48 hours
Should not drive for six weeks
May drive (but inform DVLA)
No need to inform DVLA

For each question below, choose the SINGLE most likely answer form the above list of
options. Each option may be used once, more than once, or not at all.

183. A 45-year-old IDDM with no complications or other medical problems.


184. A 35-year-old has epilepsy with attacks occurring during sleep. How long
must he wait before he can resume driving?
185. A 25-year-old lady has undergone a wisdom tooth extraction under
general anesthesia. What advice do you give her about driving?
186. A drill worker has complete loss of vision in his left eye following and
accidental injury at work. He is otherwise well and has 6/6 vision with a
normal visual field on the right.
187. A 50-year-old has undergone a coronary angioplasty. How long should he
not drive after the operation?
188. A 50-year-old lady is diagnosed to have glaucoma. Visual acuity is normal
but binocular field of vision is limited to the central 60 o only.

71

EMQs by Maju Mathews

Answers: Fitness to Drive


183. (J)
All patients on oral hypoglycemic and insulin must inform the DVLA.

184. (G)
License may be granted if free from attach for 1-year prior to license
validity or if previous 3-year attack only during sleep.

185. (H)
______

186. (J)
Monocular vision is allowed only if visual field is full.

187. (B)
Others include pacemaker, heart valve or artery surgery or uncomplicated
MI.

188. (A)
Binocular field of vision must be > 120.

72

EMQs by Maju Mathews

Theme: Management of chronic joint diseases


Options
A.
B.
C.
D.
E.
F.
G.
H.
I.

Steroids
Chloramphenicol
Sulfasalazine
Penicillin
Physiotherapy
Rifampicin
Dextropropoxyphene
INH
Methotrexate

For each question below, choose the SINGLE most likely answer form the above list of
options. Each option may be used once, more than once, or not at all.

189. A 30-year-old farmer presents with history of myalgia, fatigability, with


occasional bouts of fever, depression, and right knee-joint pain, for the
past 6 months. On examination, there is wasting of the muscles of the
hand and limited movements of the joints.
190. A 46-year-old lady with a known h/o duodenal ulcers on treatment,
develops pain, swelling and stiffness of the metacarpophalangeal, distal
interphalangeal and wrist joints, which has been progressively getting
worse. On examination, there is wasting of the muscles of the hand and
limited movements of the joints.
191. A 21-year-old man presents with episodic pain in both buttocks, low back
pain and stiffness worse in the mornings.
192. A 30-year-old lady presents with severe scaly erythematous lesions all
over her body, with swelling and severe pain on her DIP joints which is
severely disabled as she is unable to move the joint.
193. A 22-year-old homosexual man develops high-grade fever associated with
pustules on the hands with severe joint pain. He later develops swelling
and redness of the left knee.

73

EMQs by Maju Mathews

Answers: Management of Chronic Joint Diseases


189. (F)
Brucellosis; commonly seen in people handling animals.

190. (C)
As this lady has a duodenal ulcer, it is not possible to give NSAIDs

191. (E)
Exercise and Physiotherapy are the main ways of managing this condition

192. (I)
Extensive Psoriasis

193. (D)
Gonococcal infection

74

EMQs by Maju Mathews

Theme: Causes of UTI in children


Options
A.
B.
C.
D.
E.
F.
G.
H.
I.
J.
K.

Asymptomatic bacteriuria
Acute pyelonephritis
Acute glomerulonephritis
Posterior urethral valve
Sexual abuse
Urethral stricture
Spina bifida
Trauma
Ureterocele
Secondary Enuresis
Secondary to indwelling catheter

For each question below, choose the SINGLE most likely answer form the above list of
options. Each option may be used once, more than once, or not at all.

194. A 10-year-old girl brought by her mother diagnosed as UTI. This is the
fourth episode in the past 1 year. She also complains of mild bleeding per
vaginam. She has been going for horse riding classes since one year.
195. A 4-year-old boy presents with poor urinary stream and has a bloated
appearance. Investigations done show hyponatremia, and ultrasound
shows bilateral hydronephrosis and hydroureter.
196. A mother brings her 6-year-old child concerned that she is bed-wetting at
night. She says that this has started since 1 month, prior to which she
insists that bladder control was normal.
197. A 7-year-old boy with a urethral stricture diagnosed to have UTI, which
grew staphylococcus epidermidis.
198. A 12-year-old girl presents with fever and passing smoky urine since 4
days. She had recovered from a sore throat 3 weeks back. Urine
microscopy shows presence of red and white cells and culture grew
significant colony of E. coli.

75

EMQs by Maju Mathews

Answers: Causes of UTI in Children


194. (E)
Recurrent unexplained UTI in a child within a short period of time.

195. (D)
Secondary to the posterior urethral valve.

196. (J)
Typical history of secondary enuresis that can be a cause for UTI.

197. (K)
Staph. Epidermidis is the commonest organism causing UTI secondary to
indwelling catheter. The urethral stricture necessitates catheterization.

198. (C)
History suggestive of post-streptococcal glomerulonephritis.

76

EMQs by Maju Mathews

Theme: Diagnosis of peripheral neurological lesions


Options
A.
B.
C.
D.
E.
F.
G.
H.
I.
J.
K.

Median nerve lesion


Ulnar nerve lesion
Brachial plexus lesion
Cord compression
Cauda equina lesion
Autonomic neuropathy
Radial nerve lesion
Peripheral neuropathy
Multiple sclerosis (demyelination)
Anterior horn cell disease (motor neurone disease)
Sciatic nerve lesion

For each question below, choose the SINGLE most likely answer form the above list of
options. Each option may be used once, more than once, or not at all.

199. A 38-year-old pregnant woman complaints of tingling sensation in her


hands, worse at night. Her Tinels and Phalens signs were positive.
200. A 72-year-old woman presents with bilateral foot drop and spastic
weakness with muscle fasciculation of her legs. Recently, she has been
prone to having chest infections. Her tongue was wasted and fasciculating.
201. A 70-year-old diabetic man complains of impotence and intermittent
vomiting, with abdominal distension after eating. He also has postural
hypotension.
202. A 55-year-old man complains of left shoulder pain and weight loss. There
is evidence of miosis and partial ptosis of his left eye.
203. A 75-year-old man complains of bilateral lower limb weakness, numbness
and urinary incontinence. His knee and ankle reflexes are absent. There is
loss of sensation in the sacral dermatomes.

77

EMQs by Maju Mathews

Answers: Diagnosis of Peripheral Neurological Lesions


199. (A)
Carpal tunnel syndrome

200. (J)
Motor neurone disease is associated with degeneration of motor neurons,
somatic motor nuclei of cranial nerves and cortex. Confirmed by
Electromyography.

201. (F)
Diabetes with autonomic features.

202. (C)
Horners syndrome. Carcinoma of the apex of lung can erode the ribs and
lower part of the brachial plexus.

203. (E)
Compression of the cauda equina causes these symptoms. Urgent
surgery is indicated.

78

EMQs by Maju Mathews

Theme: Failure to thrive


Options
A.
B.
C.
D.
E.
F.
G.
H.
I.
J.
K.

Coeliac disease
Cystic fibrosis
Blind loop syndrome
Cows milk protein intolerance
Neglect
Hypogammaglobulinemia
Hirschsprungs disease
Hypothyroidism
Munchausen syndrome by proxy
Inborn errors of metabolism
Gastroesophageal reflux disease

For each question below, choose the SINGLE most likely answer form the above list of
options. Each option may be used once, more than once, or not at all.

204. A 3-year-old child brought with inactivity, excessive sleeping, slow feeding
and little crying. Developmental milestones are delayed. On examination: flat
nasal bridge, protruded tongue with hypotonia.
205. A 2-year-old brought with fits and persistent diarrhea. Mother describes
childs urine as having a musty odor. There is evidence of impaired
milestone development. On examination, child has light colored hair with
eczema.
206. A 5 year boy with the history of recurrent chest infections and slow growth.
He has been passing greasy foul smelling stools. On examination: child
has clubbing and abdominal distension with marked muscle wasting of the
buttocks. Investigations show increased blood glucose.
207. A 10-year-old girl presents with chronic diarrhea and vomiting. Her height
was on the 50th centile and weight on the 15th centile. She also has marked
pallor and is diagnosed to have severe iron deficiency anaemia.
208. A 6-month-old girl presents with failure to thrive. She was admitted two
weeks back with loose stools and diarrhea, which stopped with ORS
treatment. Her physical development prior to admission was normal. On
examination, baby alert and responsive and it was noticed that mother
insisted on feeding the baby herself and stayed continuously with the
baby.
209. A 10-year-old girl presents with failure to thrive and recurrent infections
needing repeated antibiotic treatment. She also complains of persistent
cough with suppurative sputum production. On examination: pallor,
clubbing, coarse crepitations and hepato-splenomegaly.

79

EMQs by Maju Mathews

Answers: Failure to Thrive


204. (H)
Common cause of failure to thrive.

205. (J)
Phenylketonuria

206. (B)
Commonest autosomal recessive disease, Diagnose with sweat test.

207. (A)
Malabsorption caused by gluten. Treat with gluten free diet.

208. (I)
Usually applied to children this term is used when a patient is thought to
be ill as a result of false history given by another individual.

209. (F)
Some form of this condition is congenital (eg. Wiskott Aldrich Syndrome)
while other develops during adult life.

80

EMQs by Maju Mathews

Theme: Choice of antibiotics


Options
A.
B.
C.
D.
E.
F.
G.
H.
I.
J.
K.
L.

None
Benzylpenicillin
Amoxicillin
Cefotaxime
Gentamicin
Metronidazole
Ciprofloxacin
Erythromycin
Trimethoprim
Rifampicin
Co-amoxiclav
Two or more Antibiotics together

For each question below, choose the SINGLE most likely answer form the above list of
options. Each option may be used once, more than once, or not at all.

210. A 17-year-old boy develops diarrhea with cramping abdominal pain after a
party. More than half the guests are similarly affected.
211. A 27-year-old nurse complains of headache, neck stiffness, fever and
purpuric rash. What antibiotic should be immediately given?
212. A 35-year-old male abattoir worker develops a lesion on his hand, which
ulcerates forming a black eschar.
213. A 5-year-old girl presents with a painful left ear. On examination, the child
is pyrexial and otoscopy reveals a bulging red eardrum.
214. A 35 -year-old woman, presents with increased frequency of micturition
and burning sensation during micturition. On examination no loin
tenderness or other signs.

81

EMQs by Maju Mathews

Answers: Choice of Antibiotics


210. (G)
Useful in shigella, salmonella, campylobacter gastroenteritis. Also used in
travelers diarrhea.

211. (B)
Empirical treatment should not be delayed if meningococcal infection is
suspected.

212. (B)
Benzylpenicillin is the treatment of choice in anthrax.

213. (C)
May need aural toilet if not responding.

214. (I)
Treatment includes drinking plenty of fluids, urinating often, doublevoiding, voiding after intercourse.

82

EMQs by Maju Mathews

Theme: Diagnosis of proximal muscle weakness


Options
A.
B.
C.
D.
E.
F.
G.
H.
I.
J.

Cushings syndrome
Diabetic amyotrophy
Non-metastatic manifestation of malignancy
Polymyositis
Hypocalcemia
Thyrotoxicosis
Duchenne muscular dystrophy
Hypokalemia
Hypothyroidism
Polymyalgia rheumatica

For each question below, choose the SINGLE most likely answer form the above list of
options. Each option may be used once, more than once, or not at all.

215. A 70-year-old man complains of 2 months history of painful thighs and


difficulty getting up from a chair. Examination revealed wasting of the
quadriceps with absent knee reflexes. His glycosylated haemoglobin is
elevated at 9%.
216. A 45-year-old woman with long standing asthma complains of obesity and
proximal muscle weakness. She requires a maintenance steroid dose of at
least 15-mg prednisolone per day.
217. A 7-year-old boy has hypertrophy of his calves and has a waddling gait.
He also displays the Gower sign, due to weakness of his spine and knees.
218. A 56-year-old man with recently diagnosed chronic renal failure complains
of paraesthesia, cramps, proximal muscle weakness and tetany. Both
Chvosteks and Trousseaus sings were elicited.
219. A 47-year-old woman presents with symmetrical proximal muscle
weakness and wasting. Her muscles are tender, with reduced tendon
reflexes. Serum creatinine phosphokinase (CPK) is elevated.

83

EMQs by Maju Mathews

Answers: Choice of Antibiotics


215. (B)
Proximal muscles commonly involved. Seen in long standing diabetics.

216. (A)
Long term use of steroids can cause Cushings syndrome with proximal
myopathy.

217. (G)
Sex linked recessive. Serum creatine kinase raised. No specific
treatment.

218. (E)
Conversion of vitamin-D to its active moiety is impaired in renal failure,
hence resulting in hypocalcemia.

219. (D)
Symmetrical proximal muscle weakness resulting from muscle
inflammation. Dysphagia and respiratory weakness may develop. A
quarter of those affected has heliotrope rash. Diagnosed by CK, EMG and
muscle biopsy.

84

EMQs by Maju Mathews

Theme: Cranial nerve defects


Options
A.
B.
C.
D.
E.
F.
G.
H.
I.
J.
K.
L.

Olfactory (I)
Optic (II)
Oculomotor (III)
Trochlear (IV)
Trigeminal (V)
Abducens (VI)
Facial (VII)
Auditory (VIII)
Glossopharyngeal (IX)
Vagus (X)
Accessory (XI)
Hypoglossal (XII)

For each question below, choose the SINGLE most likely answer form the above list of
options. Each option may be used once, more than once, or not at all.

220. A 45-year-old man with type-1 (insulin-dependent) diabetes mellitus has a


painful right eye and double vision. The pupil is enlarged and the eye
deviated inferiorly and laterally. There is also ptosis.
221. A 35-year-old lady, previously well, complains of a num feeling of the left
side of her face and a watery left eye. There is a loss of the nasolabial fold
and the left forehead creases. She is unable to whistle or show her teeth
on the left. Sensation is normal.
222. A 70-year-old lady on prednisolone for rheumatoid arthritis complains of a
painful eye and rash. There is a vesicular rash involving the right
conjunctiva and right forehead, which is very tender to touch.
223. A 72-year-old man complains of great difficulty in walking downstairs and
occasional diplopia. There are diminished eye movements on the right
side where the eye is unable to move to the inferior and medical position.
224. A 20-year-old lady presents with blurring of vision. She has a mildly dilated
right pupil and visual acuity of 6/40 in the right eye compared with 6/6 in
the left eye. There is no ptosis.

85

EMQs by Maju Mathews

Answers: Cranial Nerve Defects


220. (C)
Diabetes mellitus is the commonest cause if III-nerve palsy. Prognosis is
usually Good.

221. (G)
Can be idiopathic, or caused by cholesteatoma, Herpes zoster, or tumour
of the parotid gland.

222. (E)
Herpes zoster of the trigeminal nerve is commonly seen in the
immunocompromised.

223. (D)
A trochlear nerve defect classically presents as difficulty in walking down
stairs because there is a loss of movement in the superior oblique muscle.

224. (B)
Multiple sclerosis commonly affects the optic nerve.

86

EMQs by Maju Mathews

Theme: Specific emergency management


Options
A.
B.
C.
D.
E.
F.
G.
H.
I.
J.
K.

Check blood glucose


Hydrocortisone succinate
50% glucose
IM chlorpromazine
Pass NG tube.
Propranolol
IV access, blood sampling & IV normal saline
IV access, blood sampling IV gelatin solution
10 % Calcium gluconate
broad spectrum antibiotics
Urine cultures

For each question below, choose the SINGLE most likely answer form the above list of
options. Each option may be used once, more than once, or not at all.

225. A 45-year-old male, a known case of Addisons is brought in shock. He


had been complaining of dysuria and had a very high fever since
yesterday. A diagnosis of addisonian crisis is made. Your first action would
be.
226. He is confused and has seizures and now seems to have right
hemiparesis. Next step.
227. You have sent off the bloods and are awaiting results in the above patient
and have stabilized his vitals, your next step would be
228. Now the patient has improved, but the ECG shows broadened QRS
complexes, tall T-waves and disappearing P-waves. He is likely to
require.
229. You have to admit him now and are preparing his drug chart. You would
like to write up .
230. 45-year-old known hyperthyroid has been brought in with restlessness,
tremors, palpitations, & tachycardia. You diagnose Thyrotoxic crisis. Your
first step would be .
231. She continues to be restless and is thrashing around. Next step.
232. She is gradually beginning to settle down. Next step .

87

EMQs by Maju Mathews

Answers: Specific Emergency Management


225. (H)
Plasma expanders like gelatin solutions are preferred to normal saline.
The first step in every shocked patient is resuscitation.

226. (A)
Hypoglycemia is a common cause of these symptoms in addisonian crisis
Check blood glucose first

227. (B)
The crisis develops due to the shortage of steroids in the body. So
replacement of the same is extremely important.

228. (I)
Hyperkalemia is a common accompaniment of addisonian crisis.

229. (J)
Infection, trauma, myocardial infarction and stress are some of the
common Causes of addisonian crisis.

230. (G)
As the patient is not in shock in this case, normal saline is preferred.

231. (D)
Sedation is important.

232. (F)
Given together with carbimazole and iodine.

88

EMQs by Maju Mathews

Theme: Management of dermatological infections


Options
A.
B.
C.
D.
E.
F.
G.
H.
I.
J.

Penicillin G
Erythromycin
Topical fusidic acid
Electrocautery
Oxytetracycline
Topical Miconazole
Laser
Itraconazole
Acyclovir
No treatment required

For each question below, choose the SINGLE most likely answer form the above list of
options. Each option may be used once, more than once, or not at all.

233. An 8-year-old girl presents with brown crusting lesions on the chin. She is
otherwise doing well.
234. A 32-year-old butcher presents with purplish red lesions on the hands and
forearm. He says that it started at the site of a cut on the forearm.
235. A 30-year-old lady presents with a blistering rash on the medial aspect of
the left thigh. She had felt pain and tingling at the site prior to the onset.
236. A 49-year-old obese man presents with severe itching in his groin. On
examination, small circular areas of erythema and pustules are present in
front of an advancing edge. Raw areas are left behind when the plaques
are scrapped off.
237. A 4-year-old child presents with multiple small papules on the face. On
examination, there is central depression in the papules.

89

EMQs by Maju Mathews

Answers: Management of Dermatological Infections


233. (C)
Impetigo. Treated with local application of Fusidic acid.

234. (A)
Erysipeloid: This distinctive skin infection is caused by direct cutaneous
inoculation with Erysipelothrix rhusiopathiae. Because this organism is
most often associated with fish and domestic swine, erysipeloid most
commonly results from an occupational injury related to fishing (fishhandlers disease) or slaughterhouse work. After an incubation period of
several days, pain (often severe), edema, and a well-demarcated,
purplish-red lesion develop. Definitive diagnosis requires isolation of the
bacteria from a biopsy specimen, a tissue aspirate, or blood.
235. (I)
Herpes. Usually has a dermatomal distribution.

236. (F)
Candida, this can cause severe itching. Another treatment is topical
Clotrimazole.

237. (J)
Caused by DNA virus.

90

EMQs by Maju Mathews

Theme: Diagnosis of upper limb injuries


Options
A.
B.
C.
D.
E.
F.
G.
H.
I.
J.
K.
L.

Clavicular fracture
Humeral head/neck fracture
Colles fracture
Scaphoid fracture
Shoulder dislocation
Smiths fracture
Bennetts fracture
Carpal tunnel syndrome
Radial head subluxation
Supracondylar fracture
Radial head fracture
Dislocated elbow

For each question below, choose the SINGLE most likely answer form the above list of
options. Each option may be used once, more than once, or not at all.

238. A 20-year-old male had a fall on outstretched hand today. Reduced


movement at right wrist. Anatomical snuffbox is tender. X-ray of the wrist is
normal.
239. A 68-year-old lady with fall on outstretched hand. She has painful left
wrists. On examination, wrist is unduly prominent on the dorsal aspect with
radial deviation.
240. A 2-year-old child was brought to the A&E with inability to use the left
upper limb, which followed his dad lifting him up, by the wrist while playing.
On examination his elbow is flexed and pronated.
241. A 25-year-old male with fall on outstretched hand. Extension and flexion is
limited, pronation and supination in normal. Elbow is swollen with fat pad
sign on X-ray.
242. A 10-year-old had a fall on outstretched hand now has pain in right elbow.
On examination the elbow is swollen and deformed. The relationship
between the olecranon and condyles is preserved.

91

EMQs by Maju Mathews

Answers: Diagnosis of Upper Limb Injuries


238. (D)
Is very easy to miss on X-rays and should be repeated after 2 weeks.

239. (C)
Common in osteoporotic postmenopausal females

240. (I)
Radial heads slips out of the annular ligament.

241. (K)
X-ray often shows an effusion and minor fractures are often missed.

242. (L)
The elbow should be kept in extension after the injury to prevent damage
to the brachial artery.

92

EMQs by Maju Mathews

Theme: Opportunistic infections in HIV


Options
A.
B.
C.
D.
E.
F.
G.
H.
I.
J.

Molluscum contagiosum
Hairy leukoplakia
Tuberculosis
Cryptosporidium
CMV infection
Candida infection
Pneumocystis carinii infection
Cryptococcal infection
Toxoplasma abscess
Aids dementia

For each question below, choose the SINGLE most likely answer form the above list of
options. Each option may be used once, more than once, or not at all.

243. A patient presents with diminishing visual acuity and headache.


Fundoscopy shows large, multiple cotton wool spots in both eyes.
244. A patient presents with gradual onset of fever and headache with impaired
level of consciousness. CSF shows no organisms on gram stain. CT brain
is normal.
245. A patient complains of fever, dry cough and shortness of breath. He is
tachypneic but chest is clear on examination. Oxygen saturation shows
hypoxemia.
246. A patient presents with profuse watery diarrhea, abdominal pain, nausea
and vomiting for past one month. He has tried several antidiarrheals
without any relief in his symptoms.

93

EMQs by Maju Mathews

Answers: Opportunistic Infections In HIV


243. (E)
CMV retinitis is seen in 45% of patients and can cause as decreased
visual acuity or blindness.

244. (H)
Causes insidious meningitis in up to 8% of patients.

245. (G)
Diagnosed by bronchoalveolar lavage, induced sputum or lung tissue
microscopy.

246. (D)
Fungal infection causing infection in 10-50% of patients.

94

EMQs by Maju Mathews

Theme: Disorders affecting the mouth


Options
A.
B.
C.
D.
E.
F.
G.
H.
I.
J.
K.
L.
M.
N.

Leucoplakia
Lichen planus
Aphthous ulcers
Steven-Johnson syndrome
Psoriasis
Iatrogenic gingivitis
Candidiasis
Bechets disease
Pemphigoid
Peutz-Jeghers syndrome
Addisons disease
Squamous cell carcinoma
Osler-Weber-Rendu Syndrome
Folate deficiency

For each question below, choose the SINGLE most likely answer form the above list of
options. Each option may be used once, more than once, or not at all.

247. A 36-year-old female, known epileptic presents to her GP with painful


swollen gums.
248. A 60-year-old male smoker presents with a painful single indurated ulcer
with rolled out edges in his right cheek.
249. This 56-year-old female had been treated for a UTI with amoxicillin by her
GP. She has now developed fever, arthralgia and has vesicular eruptions
in her mouth.
250. A 40-year-old man presents with abdominal pain and rectal bleeding. He is
noted to have pigmented papules on his lips.
251. A 48-year-old male presents with purplish papules around his wrist that
are intensely itchy. He also has lace like lesions in his mouth.
252. A 28-year-old female present sudden loss of vision in her right eye. A
diagnosis of central retinal vein occlusion is made. She has previously has
recurrent attacks of painful red eye with photophobia. She has been
having recurrent painful ulcers on the labia. She has been administered a
variety of treatments without success for the genital ulcers.

95

EMQs by Maju Mathews

Answers: Disorders affecting the Mouth


247. (F)
Caused by phenytoin.

248. (L)
Squamous cell CA of the cheek is seen in smokers, heavy drinkers, those
who chew pan and in those who store plug inside their cheeks.

249. (D)
Stevens Johnson syndrome or erythema multiforme major presents with
the above symptoms and may occur as a result of drugs (penicillins,
sulfonamides and some sedatives), viral infections (orf, herpes),
neoplasms or other systemic diseases.
250. (J)
Autosomal dominant condition. Peutz-Jeghers syndrome comprises of
intestinal polyposis and black freckles on lips, oral mucosa and other
areas. There may be massive GI bleeding. Gastric & duodenal polyps are
premalignant.
251. (B)
LP usually presents in young adults and in middle age as purplish papular
pruritic rashes mainly involving the writs, flexor aspects of the forearm,
genitals, lumbar regions, ankles, there may be lace like lesions in the oral
cavity. The glans may have annular white lesions. Wickmans striate and
Koebners phenomenon present. Resolves spontaneously within a year of
onset. Betamethasone valerate cream may help.
252. (H)
Bechets disease presents with recurrent painful ulcers of the mouth,
scrotum and labia, arthritis, iritis, keratitis, hypopyon, CRVO,
meningoencephalitis, Parkinsonism, dementia and thrombophlebitis.
Steroid cream for ulcers.

96

EMQs by Maju Mathews

Theme: Diagnosis of hypercalcemia


Options
A.
B.
C.
D.
E.
F.
G.
H.
I.
J.
K.
L.

Hyperparathyroidism
Milk-alkali syndrome
Sarcoidosis
Thyrotoxicosis
Multiple myeloma
Small cell lung carcinoma
Squamous lung carcinoma
Bone metastases
Immobility
Thiazide diuretics
Vitamin-D toxicity
Pagets disease

For each question below, choose the SINGLE most likely answer form the above list of
options. Each option may be used once, more than once, or not at all.

253. A 28-year-old marathon runner presents with nausea for about 2 months.
He is otherwise fit and well and not on any prescribed medication,
although he regularly takes various dietary and mineral supplements.
254. A 50-year-old ex-smoker presents with weight loss and thirst. He has
clubbing and has right middle lobe consolidation demonstrated in his chest
X-ray.
255. A 46-year-old woman develops progressively worsening breathlessness
over the last 1 year. Chest X-ray shows reticulonodular shadowing and
she was incidentally found to have raised calcium of 2.97 mmol/I.
256. A 70-year-old deaf woman presents to A&E following a minor fall. On
examination she is found to have a deformity in her left leg and X-ray
reveals fracture tibia with osteosclerotic changes. She was also found to
have a slightly raised calcium.
257. A 73-year-old man presents with severe back pain. X-ray pelvis reveals
osteolytic lesions. His erythrocyte sedimentation rate (ESR) is elevated at
85-mm/hour.
258. A 65-year-old woman present with increasing thirst and pain over the
lumbar region. She is on Tamoxifen following a mastectomy for CA breast,
which she had undergone 2 years back.

97

EMQs by Maju Mathews

Answers: Diagnosis of Hypercalcemia


253. (K)
Vitamin D toxicity is usually iatrogenic.

254. (G)
Clues in history are ex-smoker, weight loss.

255. (C)
Think of sarcoidosis in anyone presenting with gradually and
breathlessness and the characteristic bilateral hilar lymphadenopathy on
chest X-ray. Raised calcium is due to increased Vitamin D turnover.

256. (L)
Characteristic presentation of Pagets disease.

257. (E)
----------------------

258. (H)
Secondary bone deposits, especially over lumbar vertebrae following
breast carcinoma in not uncommon and should be considered especially
in association with Calcium levels.

98

EMQs by Maju Mathews

Theme: Diagnosis of hyponatremia


Options
A.
B.
C.
D.
E.
F.
G.
H.
I.
J.
K.

Addisons disease
Nephrotic syndrome
Diuretics
Pseudohyponatremia
Artefactual
Syndrome of inappropriate anti-diuretic hormone secretion
Dilutional hyponatremia
Chronic liver disease
Hypothyroidism
Oliguric renal failure
Excessive sweating

For each question below, choose the SINGLE most likely answer form the above list of
options. Each option may be used once, more than once, or not at all.

259. A 16-year-old girl comes to A&E with confusion and fits. On examination
she has a bloated appearance with pedal oedema. She is a known case of
obsessive compulsive disorder with a history of drinking around 10 liters in
the past 24 hours.
260. A 23-year-old diabetic lady on insulin complains of dizziness on standing
form a sitting/lying position. She is pigmented with significant postural
hypotension. Biochemistry investigations show serum Na is 128-mmol/I.
261. A 37-year-old male known case of IDDM is brought to A&E with confusion
and stupor. Ward glucose test shows 22.3-mmol/I and urine dipstick shows
presence of ketones. His serum sodium is 115-mmol/liter.
262. A 65-year-old male with a diagnosis of CA-bronchus presents with
nausea, confusion and irritability. His plasma Na level are 114-mmol/l.
263. A 55-year-old woman, a known case of SLE is found to have oedema of
her legs extending up to her thighs, and a serum sodium of 125-mmol/l.

99

EMQs by Maju Mathews

Answers: Diagnosis of Hyponatremia


259. (G)
A case of psychogenic polydipsia leading to dilutional hyponatremia.

260. (A)
Patient has IDDM with postural hypotension, pigmentation
hyponatremia other electrolyte abnormality is hyperkalemia.

&

261. (D)
Patient with diabetic ketoacidosis.

262. (F)
SIADH may occur.

263. (B)
Nephrotics syndrome may occur in SLE leading to oedema and
hyponatremia.

100

EMQs by Maju Mathews

Theme: Complications of fractures


Options
A.
B.
C.
D.
E.
F.
G.
H.
I.
J.

Delayed union
Non union
Pseudoarthrosis
Malunion
Sudecks atrophy
Contractures
Secondary infection
Compartment syndrome
Fat embolism
Myositis ossificans

For each question below, choose the SINGLE most likely answer form the above list of
options. Each option may be used once, more than once, or not at all.

264. A 55-year-old female presents to A&E a month after a radial head fracture.
She has been treated for the condition and a recent x-ray revealed good
healing. Now she complains of swelling of the hand & is very sensitive to
touch. The hand appears and cyanosed.
265. A 36-year-old man was admitted with a femoral shaft fracture 4 days back.
He has well when he complained of sudden onset of difficulty in breathing.
He had seizures and went into coma. Examination revealed diffuse
petechial rashes.
266. This 65-year-old lady who was treated for Colles fracture six months back
is unable to use a can opener.
267. A 10-year-old child was treated for a right supracondylar fracture. He
presents 4 weeks later with poor range of movement at the elbow, which
worsened with physiotherapy. X-ray revealed a faint area of new bone
formation with indistinct edges.
268. A 52-year-old man was diagnosed with right neck of femur fracture 4
months back. He still continues to have pain, swelling and reduced
mobility. X-ray reveals closing off of the medullary cavity.

101

EMQs by Maju Mathews

Answers: Complications of Fractures


264. (E)
Loss of vascular tone and supersensitivity to sympathetic neurons.

265. (I)
Characteristically occurs 3-10 days after the fracture (fat embolism)

266. (D)
Recognized complication of Colles fracture.

267. (J)
Calcification of particular haematoma leading to restriction of movement
and loss of function.

268. (B)
Commoner in cortical bone.

102

EMQs by Maju Mathews

Theme: Pediatrics-respiratory disorders


Options
A.
B.
C.
D.
E.
F.
G.
H.

Tracheoesophageal fistula
Choanal atresia
Bronchiolitis
Croup
Acute epiglottitis
Asthma
Hypothyroidism
Whooping cough

For each question below, choose the SINGLE most likely answer form the above list of
options. Each option may be used once, more than once, or not at all.

269. A 7-year-old presents with fever, running nose, cough, nasal flaring and
hyper-expansion of the chest. Auscultation reveals expiratory wheezes.
270. A 6-month-old infant presents with fever, aspiratory stridor and flaring of
the nostrils. The mother says that the child has had an upper respiratory
infection for the past 2 days.
271. A 5-year-old child wakes up at night with high fever, sore throat,
respiratory distress, drooling and stridor.
272. A 3-year-old child presents with bouts of coughing which end in vomiting.
These episodes are worse at nigh and after feeds and are often
associated with cyanosis. The child is afebrile.
273. A newborn infant has excess drooling, with coughing and choking during
feeds. Cyanosis is present which is unrelieved by crying.

103

EMQs by Maju Mathews

Answers: Pediatrics-Respiratory Disorders


269. (C)
Commonest lower respiratory tract infection of infancy. Caused by
respiratory syncytial virus.

270. (D)
Caused by parainfluenza virus.

271. (E)
Diagnosed by laryngoscope. Lateral neck X-rays shows enlarged glottis.
Caused by H influenzae. It is an emergency.

272. (H)
Caused by Bordetella pertussis. No wheeze

273. (A)
Treat with surgical correction of the fistula.

104

EMQs by Maju Mathews

Theme: Symptoms and signs of cranial nerve lesions


Options
A.
B.
C.
D.
E.
F.
G.
H.
I.
J.
K.
L.

1st cranial nerve


2nd cranial nerve
3rd cranial nerve
4th cranial nerve
5th cranial nerve
6th cranial nerve
7th cranial nerve
8th cranial nerve
9th cranial nerve
10th cranial nerve
11th cranial nerve
12th cranial nerve

For each question below, choose the SINGLE most likely answer form the above list of
options. Each option may be used once, more than once, or not at all.

274. Patient presented with headache and vomiting. On examination, patients


jaw moves to the right when opened. There is reduced facial sensation on
the left with loss of corneal reflex.
275. A 65-year-old patient was walking in the garden when he noticed a
drooping and weakness of the left side of the face and he is unable to
raise the left eyebrow.
276. A patient complains of impairment of taste and smell. On formal testing,
sense of smell is lost and taste is mildly reduced.
277. 74-year-old presents with headache and dizziness and instability of gait.
The patients tongue deviates to the left when protruded. The patient
cannot easily move the tongue from side to side.
278. 32-year-old male with IDDM presents with diplopia on looking to the right
and is unable to abduct the right eye.
279. A patient has difficulty swallowing. There is impaired movement of the
palate, which is pulled to the left on saying Ah. Gag reflex is absent.

105

EMQs by Maju Mathews

Answers: Symptoms and Signs of Cranial Nerve Lesions


274. (E)
The trigeminal nerve has both sensory and motor components. The nerve
transmits sensation from the face, mouth, lips, eyes, forehead and
anterior part of the scalp as well as the dura of the anterior cranial and
middle fossa. The three branches of the nerve are (a) ophthalmic division,
(b) maxillary division and the mandibular division. Motor functions include
supply to the muscles of mastication (the masseters, temporalis, medial
and lateral pterygoids, anterior belly of the diagastric, mylohyoid, tensor
veli palatini and tensor tympani muscles).
275. (G)
The facial nerve supplies the muscles of the face. In upper motor lesions,
there is sparing of the area of the face above the eyebrows. So in this
case, he has a lower motor neuron lesion of the facial nerve, which in the
absence of any underlying lesion is called bells palsy.
276. (A)
Olfactory nerve lesion.

277. (L)
In upper motor hypoglossal nerve lesions, there is an inability to move the
tongue. The tongue deviates to the affected side as the muscle of the
healthy side pushes its.
278. (F)
The lateral rectus is supplied by the 6th cranial nerve, the lesion of which
gives these signs including diplopia worse on looking to the side of the
paretic LR. Lesion associated with diabetes.

279. (J)
The vagus nerve is the predominant motor supply to the upper pharynx,
soft palate and the intrinsic muscles of the larynx and the cricothyroid.

106

EMQs by Maju Mathews

Theme: Knee injuries


Options
A.
B.
C.
D.
E.
F.
G.
H.
I.

Patellar fracture
Dislocation of patella
Posterior cruciate ligament rupture
Anterior cruciate ligament rupture
Collateral ligament injury
Ruptured patellar tendon
Osgood-Schlatters disease
Meniscal injury
Osteochondritis desiccans

For each question below, choose the SINGLE most likely answer form the above list of
options. Each option may be used once, more than once, or not at all.

280. A 45-year-old lady who was hit in the back of her knee by a car has a
swollen knee and the inability to flex the knee. The knee repeatedly gives
way. Anterior draw test is positive.
281. A 12-year-old boy has been having recurrent right knee pain. He presents
to A&E with increased pain today. He played football a few hours back. He
didnt injure himself. On examination, right tuberosity is prominent with
tenderness in the lower one third of the patellar tendon. X-ray shows
enlarged and fragmented tuberosity.
282. A 30-year-old footballer kicked the ball with his right foot and soon after
experienced sharp pain in the left knee. Left knee extension is limited and
he is unable to weight bear. X-ray of left knee is normal.
283. A 35-year-old male presents following a fall on the knee, which is held in
flexion. There is no knee swelling or signs of ligament injury. X-ray is
normal.
284. A 20-year-old with recurrent pain and swelling of the knee, which is,
increased post exercise. The knee is locked.

107

EMQs by Maju Mathews

Answers: Knee Injuries


280. (D)
Typically follows posterior blows to the tibia or rotation injuries when the
foot is fixed to the ground.

281. (G)
Anterior knee pain after exercise is characteristic.

282. (H)
Medial meniscal tears follow forced twists to a flexed knee.

283. (B)
Interpretation can be difficult as the patella overlies the distal femur on AP
view and can obscure subtle fractures.

284. (I)
Local necrosis of the articular cartilage and its underlying bone.

108

EMQs by Maju Mathews

Theme: Diagnosis of endocrine conditions


Options
A.
B.
C.
D.
E.
F.
G.
H.
I.
J.
K.

Conns Syndrome
deQuervains thyroiditis
Benign parathyroid adenoma
Cushings syndrome
Addisons disease
Reidels thyroiditis
Hashimotos thyroiditis
Secondary hyperaldosteronism
Follicular adenoma
Plummer disease
Diabetes mellitus

For each question below, choose the SINGLE most likely answer form the above list of
options. Each option may be used once, more than once, or not at all.

285. A 55-year-old lady with multinodular thyroid and hyperthyroidism.


286. A 55-year-old woman has hypercalcemia found as an incident finding.
Physical examination is normal in all other respects.
287. A 40-year-old male admitted with cardiac failure. Biochemistry shows
hypokalemic metabolic alkalosis and low plasma renin. He was on
treatment with diuretics for control of his BP.
288. A mother brings her 4-year-old daughter worried that the child has
persistent fever and lethargy since 2 weeks. She also complains of
frequent wet nappies and has noticed weight loss in site of the child
eating well.
289. A 28-year-old post partum lady develops a painful goitre. Her ESR is
raised and there is no radioiodine uptake on the scan.

109

EMQs by Maju Mathews

Answers: Diagnosis of endocrine conditions


285. (J)
Single or multiple toxic nodules

286. (C)
Common cause of hyperparathyroidism

287. (H)
Excess aldosterone as a result of high renin. Caused by renal artery
stenosis. Accelerated hypertension, heart failure and hepatic failure.

288. (K)
Common IDDM.

289. (F)
Rare, diagnosis can be made with certainty only by biopsy.

110

EMQs by Maju Mathews

Theme: Anticoagulants
Options
A.
B.
C.
D.
E.
F.
G.

Heparin
Warfarin
Aspirin
Ibuprofen
Greenfield
Vena caval ligation
Phenindione

For each question below, choose the SINGLE most likely answer form the above list of
options. Each option may be used once, more than once, or not at all.

290. Standard of care in prophylaxis against DVT after surgery


291. Initial treatment for patient diagnosed with DVT.
292. Recommended for polytrauma with pelvic fracture
293. Drug that inhibits platelet aggregation
294. Anticoagulant in patients with known sensitivity to warfarin.

111

EMQs by Maju Mathews

Answers: Anticoagulants
290. (A)
Low molecular weight heparin till patient is mobile.

291. (A)
Acts fast and is used as initial therapy to cover the time before warfarin to
effective.

292. (E)
No explanation needed.

293. (C)
Vital role in unstable angina, MI and post CABG

294. (G)

112

EMQs by Maju Mathews

Theme: Anatomical basis of stroke syndromes


Options
A.
B.
C.
D.
E.
F.
G.
H.
I.

Anterior cerebral artery


Dominant middle cerebral artery
Posterior cerebral artery
Vertebrobasilar system
Posterior inferior cerebellar artery
Non-dominant middle cerebral artery
Brainstem
Cerebellum
Lacunar infraction

For each question below, choose the SINGLE most likely answer form the above list of
options. Each option may be used once, more than once, or not at all.

295. A patient has weakness of the right arm and side of the face. There is
sensory neglect of that side and the patient is unable to dress or wash.
296. A patient has left homonymous hemianopia but no other defects.
297. A 64-year-old known hypertensive patient presents with mild hemisensory
impairment.
298. A patient has weakness and reduced sensation in the right leg. The arm is
mildly affected also.
299. A 64-year-old presents with sudden bilateral blindness, vertigo, dysphagia
and drop attacks. Examination reveals left-sided weakness.
300. A 52-year-old surgeon presents with vomiting, vertigo and ataxia.
Exanimation reveals ipsilateral facial numbness, diplopia, nystagmus and
contralateral sensory loss.

113

EMQs by Maju Mathews

Answers: Anatomical basis of stroke syndromes


295. (F)
Left sided parietal lesion.

296. (C)
When isolated, may be a part of the posterior circulation syndrome. The
lesion is in the optic tract and beyond. (optic tract, radiations, temporal
lobe, posterior, parietal lobe).
297. (I)
Lacunar infarcts are seen in hypertensives and consist of small infarcts in
the region of the internal capsule (partial hemiparesis or sensory loss)
pons (ataxia of cerebellar type, partial hemiparesis). Basal ganglia and
thalamus. They are often multiple. Lacunae are often thought to be
caused by the occlusion of small branch arteries or by rupture of CharcotBouchard aneurysms producing a small aneurysm, which resolves leaving
an area of infarction.
298. (A)
Areas supplied by the ACA includes the frontal lobes and the medial
cerebral hemispheres with the exception of the visual cortex of the
occipital lobes. Cortical areas include motor and sensory areas of the
lower limbs, a micturition center and supplementary motor cortex.
Ischemia of the territory of one ACA produces weakness and sensory
deficits in the opposite limbs. Some have mild transient aphasia.
299. (D)
Vertebrobasilar lesions are characterized by vertigo, ataxia, drop attacks,
dysphagia (at least two of these should occur together). There may be
bilateral or alternating weakness or sensory symptoms and sudden
blindness in patient aged over 40 years.
300. (E)
AKA Wallenberg syndrome. Symptoms: Vomiting, vertigo, ipsilateral
ataxia, contralateral pain, & temperature loss. Ipsilateral sign: facial
numbness (V-CN, diplopia (VI-CN), nystagmus, ataxia, Horners
syndrome, IX & X-CN lesions. Contralateral signs: Spinothalamic sensory
loss and rarely, mild hemiparesis.

114

EMQs by Maju Mathews

Theme: Tumour Markers


Options
A.
B.
C.
D.
E.
F.

Prostate specific antigen (PSA)


Alpha-fetoprotein (AFP)
Carcinoembryonic antigen (CEA)
CA-125
Urinary metanephrines
B-2 microglobulin

For each question below, choose the SINGLE most likely answer form the above list of
options. Each option may be used once, more than once, or not at all.

301. Ovarian carcinoma.


302. Hepatocellular carcinoma
303. Prostate cancer
304. Phaeochromocytoma
305. Colon cancer

115

EMQs by Maju Mathews

Answers: Tumour Markers


301. (D)
Tumour markers are not diagnostic of a particular disease. They are more
commonly used as a prognostic indicator.

302. (B)
Also raised in choriocarcinoma, testicular teratoma and germ cell tumors
of the ovary and is useful in the antenatal diagnosis on various conditions.

303. (A)
<10 ng/ml has a better outcome & > 20 a poor outcome.

304. (E)
24-hrs vanillylmandelic acid, metadrenaline or normetadrenaline

304. (C)
Useful in following the progression but is not of diagnostic value.

116

EMQs by Maju Mathews

Theme: Management of fractures


Options
A.
B.
C.
D.
E.
F.
G.
H.

IV antibiotics and dressing


POP Backslap
Mallet splint
Scaphoid cast and review in a fortnight
No treatment required
Manipulation under anesthesia
Analgesics only
Open reduction and internal fixation

For each question below, choose the SINGLE most likely answer form the above list of
options. Each option may be used once, more than once, or not at all.

306. A 65-year-old female fell in her bottom from a height of 3-feet.


Examination including PR is normal. The x-ray shows a fractured coccyx.
307. A 35-year-old male was playing hockey when he was hit in his right shin
by an opponent. X-ray shows a thin transverse fracture of the right tibia.
308. An 82-year-old had a fall on outstretched left hand. X-ray shows colles
fracture with the radius at a 5 degree upward tilt. Your action would be.
309. A 70-year-old presents with difficulty in extending the distal
interphalangeal joint of the little finger of her right hand with mild flexion of
the joint. She noticed this after she prepared the bed in the morning.
310. A 22-year-old male is brought to A&E with a swollen right forearm after she
was involved in a drunken brawl in the pub. Examination shows normal
movements. X-ray shows fracture of the shaft of ulna, which shows
angulation and displacement.

117

EMQs by Maju Mathews

Answers: Management of fracture


306. (G)
Unless grossly displaced no active treatment required.

307. (B)
Comminuted or segmental fracture may require open reduction and
internal fixation.

308. (F)
If involves the non-dominant hand in the very old can be ignored if the
displacement is not significant.

309. (C)
Often involves forced flexion of the DIPJ, however in the elderly can result
from minimal trauma.

310. (H)
Isolated ulnar fracture is commonly seen as a defense injury.

118

EMQs by Maju Mathews

Theme: Muscle Weakness


Options
A.
B.
C.
D.
E.
F.

Guillain Barre Syndrome


Multiple sclerosis
Fibromyalgia
Polymyalgia rheumatica
Polymyositis
Paraneoplastic syndrome

For each question below, choose the SINGLE most likely answer form the above list of
options. Each option may be used once, more than once, or not at all.

311. A 72-year-old male with adenocarcinoma of the lung presents with


proximal muscle weakness and raised creatinine kinase.
312. A 44-year-old man presents with fatigue, low-grade fever and weakness
and can no longer climb stairs to his first floor flat.
313. A 56-year-old male with recent upper respiratory infection presents with
progressive weakness in the lower extremities. He has diminished deep
tendon reflexes. His CSF protein concentration is elevated.
314. A 38-year-old woman complains of hurting all over. She sleeps well but
does not feel refreshed in the morning. She has multiple sore spots all
over the body.
315. A 64-year-old lady complains of fatigue, morning stiffness, and inability to
comb her hair because of arm weakness and an inability to get up from a
chair without assistance. Her ESR is raised.

119

EMQs by Maju Mathews

Answers: Muscle Weakness


311. (F)
Presentations include myasthenic syndrome, neuropathy, myopathy,
dermatomyositis or cerebellar degeneration.

312. (E)
Symmetrical proximal muscle weakness resulting from muscle
inflammation. Diagnose by muscle enzyme levels, EMG & muscle biopsy.
Treat with steroids.

313. (A)
Motor polyneuropathy following different infections. Paralysis
progressive and ascending. Most recover in 3 to 6 months.

is

314. (C)
It is a common diagnosis made in rheumatology clinic in patients with pain
and no obvious cause.

315. (D)
Aching and morning stiffness in proximal limbs for at least a month, also
polyarthritis, depression, fatigue, fever and anorexia.

120

EMQs by Maju Mathews

Theme: Abnormalities of the jugular venous pressure (JVP)


Options
A.
B.
C.
D.
E.
F.
G.
H.
I.

Raised JVP, normal waveform


Raised JVP, no pulsation
Large a waves
Cannon a waves
Absent a waves
Systolic (cv) waves
Slow y decent
High JVP, rises on inspiration
Normal

For each question below, choose the SINGLE most likely answer form the above list of
options. Each option may be used once, more than once, or not at all.

316. A 20-year-old female is an untreated case of rheumatic fever. She


presents with dyspnea, and an irregularly irregular pulse.
317. A 29-year-old heroin addict presents with high-grade fever with chills and
pedal oedema and abdomen and has a pulsatile, tender enlarged liver.
318. A 40-year-old woman who was previously on combined oral contraceptive
pills develops sudden on set of severe chest pain and breathlessness.
O/E, pulse is 124/min with ankle oedema. She is cold and clammy. Chest
examination is unremarkable.
319. A 68-year-old male complaining of chest pain with vomiting and sweating.
ECG shows ventricular tachycardia.
320. A 55-year-old male, heavy smoker for 35 years presents with shortness of
breath, severe cough, dysphagia and a history of blackouts, and facial
oedema.
321. A 56-year-old male, known case of chronic renal failure on hemodialysis
presents with breathlessness and collapse. He has a weak pulse 120/min,
BP 90/50. Heart sounds are faint with low voltage QRS-complexes on
ECG.

121

EMQs by Maju Mathews

Answers: Abnormalities of the Jugular Venous Pressure (JVP)


316. (E)
Rheumatic fever leading to mitral stenosis leading to atrial fibrillation with
is the only case for absent a waves.

317. (F)
Tricuspid regurgitation is common secondary to endocarditis (iv drug
abuse). Other causes being carcinoid syndrome, rheumatic and
congenital (Ebsteins anomaly). CV waves occur during ventricular
systole.
318. (A)
Combined oral contraceptives have been shown to increase the chances
of pulmonary embolism leading to right heart strain and failure. Other
causes for raised JVP with normal waveform being fluid overload and
bradycardia.
319. (D)
Cannon waves (large a wave with rapid fall) are caused due to atrial
systole against a closed ventricular valve. Other causes being complete
heart block, single ventricular pacing, atrial flutter, single chamber
ventricular pacing, nodal rhythm and ventricular extrasystole.
320. (B)
Smoking predisposes to SVC obstruction.

321. (H)
Kussmauls sing: High plateau of JVP, which raises on inspiration with
deep x and y descents.

122

EMQs by Maju Mathews

Theme: Acute treatment of arrhythmias


Options
A.
B.
C.
D.
E.
F.
G.
H.
I.
J.
K.
L.

None
Amiodarone
Digoxin
Adenosine
Beta-blockers
Verapamil
Flecainide
Lignocaine
Magnesium
Synchronized cardioversion
Pacemaker
Carotid sinus massage

For each question below, choose the SINGLE most likely answer form the above list of
options. Each option may be used once, more than once, or not at all.

322. A 65-year-old man presents with a two-hour history of severe chest pains
and palpitations. He has an irregular pulse and ECG shows SVT with a
rate of 250/min. He suddenly loses consciousness. BP is 80/30 and still is
SVT.
323. A 36-year-old man presents with palpitations for the first time. ECG shows
a narrow complex tachycardia with a rate of 190/min. He is otherwise well
and has normal blood pressure.
324. The first treatment you tried for this man was ineffective, what would you
like to try now?
325. A 70-year-old female presents to A&E with a chest infection. She has an
irregularly irregular pulse. ECG shows AF at 80/ min.
326. A 60-year-old man is admitted to Coronary Care Unit for treatment of an
inferior myocardial infarction. Two hours later he develops ventricular
tachycardia. He feels giddy but BP is normal.
327. A 70-year-old man is receiving Streptokinase for a lateral myocardial
infarction. He develops VT with central chest pain and BP 80/50. He
suddenly goes pulseless and arrests.
328. A 72-year-old man is brought to A&E with transient chest pain and
palpitations. He is hemodynamically stable. ECG shows AF at 140/min.

123

EMQs by Maju Mathews

Answers: Acute Treatment of Arrhythmias


322. (J)
All narrow complex tachycardias above the rate of 200/min, impaired
consciousness, systolic BP <90, chest pain, heart failure are to be treated
with synchronized cardioversion. If the rate is lower in the absence of
these criteria, he may be treated with adenosine.
323. (L)
As above. As the patient is younger in this case, vagal maneuvers may be
tried first.

324. (D)
As above
325. (A)
Asymptomatic AF requires no treatment.

326. (H)
If the patient is conscious and not hemodynamically compromised, he
may be treated with lignocaine. If pulse is present with adverse signs
(systolic BP <00 mm Hg, chest pain, heart failure, rate >150),
synchronized DC shock administered.
327. (J)
Same as previous answer.

328. (C)
If the arrhythmia is known to be an AF, in the absence of adverse sings,
may be treated with digoxin.

124

EMQs by Maju Mathews

Theme: Benign Breast Disease


Options
A.
B.
C.
D.
E.
F.
G.
H.
I.

Mastitis neonatorum
Lactational breast abscess
Cystic disease
Duct ectasia
Fat necrosis
Non-lactational abscess
Non-cyclical breast pain
Benign mammary dysplasia
Fibromatosis

For each question below, choose the SINGLE most likely answer form the above list of
options. Each option may be used once, more than once, or not at all.

329. A 42-year-old premenopausal lady presents with a discreet smooth breast


lump to her GP. Aspiration reveals a yellow green fluid. It is not
bloodstained she also has a mammogram and ultrasound and the lump is
no longer present.
330. A one month following the delivery of her first child, a 24-year-old lady
presents with a hot red tender swollen lump in her left breast.
331. A 50-year-old lady describes an intermittent clear, cheese-like discharge
with occasional blood staining from her breast. Her nipple has become
retracted. Fine needle aspiration cytology reveals benign cells &
mammogram reveals coarse calcification.
332. A 26-year-old lady with no family history of breast cancer presents with a
small lump in the upper outer quadrant of her breast, which is surgically
removed. Histological exam: reveals proliferation of myelofibroblasts.
333. A 40-year-old lady presents with marked premenstrual breast nodularity
and discomfort of the upper outer quadrant. No malignancy is found on
fine needle aspiration cytology. A core-biopsy reveals multiple cysts,
fibrosis and epitheliosis.

125

EMQs by Maju Mathews

Answers: Benign Breast Disease


329. (C)
Most common in the last decade of reproductive life. If aspiration
collapses the cyst and the aspirate is not blood stained, no further
treatment required.

330. (B)
Usually due to staphylococcus aureus. If early cellulitic stage, treat with
antibiotics. Any abscess to be drained.

331. (D)
Dilatation of breast ducts with periductal inflammation. May lead to
periductal mastitis or abscess or fistula. Fibrosis eventually leads to nipple
retraction. Associated with smoking. Nipple discharge may be any color,
usually watery of grumous. Try antibiotics. Usually surgery required.

332. (I)
-------

333. (H)
-------

126

EMQs by Maju Mathews

Theme: Treatment of peptic ulceration


Options
A.
B.
C.
D.
E.
F.
G.
H.
I.

Truncal vagotomy
Highly selective vagotomy
Gastroenterostomy
Underrunning
Selective vagotomy
Nissen fundoplication
Total gastrectomy
Proton pump inhibitors
Omental patch

For each question below, choose the SINGLE most likely answer form the above list of
options. Each option may be used once, more than once, or not at all.

334. A 60-year-old man attends the clinic with a long history of peptic ulcer
disease. He is dependent on NSAIDs for pain relief. He has been treated
with H2 antagonists and proton pump inhibitors without success. He is
now unable to tolerate the pain.
335. A 39-year-old executive who complains of a short history of epigastric
pain. Upper GI endoscopy shows a small duodenal ulcer. Gastric biopsy
for H. Pylori is negative.
336. An 80-year-old lady presents with hematemesis. She is hypotensive and
despite resuscitation, her BP continues to fall. Endoscopy reveals an
actively bleeding duodenal ulcer. Attempts at injection with adrenaline fail.
337. A 29-year-old man previously fit and well, presents to the casualty
department with sudden onset of epigastric pain. On examination he has
board-like rigidity of the abdomen and absent bowel sounds. Erect chest
X-ray reveals a cresenteric area of free gas beneath the left
hemidiaphragm.
338. A 60-year-old man had adequate symptom relief with medical
management of his duodenal ulcer. He complains of a bloated feeling and
starts to vomit profusely. He becomes dehydrated and is admitted for
rehydration and nasogastric aspiration. The drainage is high in volume
and a contrast X-ray reveals a tight stenosis around the site of his
duodenal ulcer.

127

EMQs by Maju Mathews

Answers: Treatment of Peptic Ulceration


334. (B)
Also called proximal gastric vagotomy. Preserves the innervation of the
pylorus.

335. (H)
Since there is no H. pylori, treat with the proton pump inhibitors.

336. (D)
Aim of the operation is to stop the bleeding. Gastrotomy & Underrunning.

337. (I)
If due to a gastric ulcer, take a biopsy to rule out malignancy.

338. (C)
Pyloric stenosis is a complication of duodenal ulcer & surgical correction
is indicated.

128

EMQs by Maju Mathews

Theme: Peripheral vascular disease


Options
A.
B.
C.
D.
E.
F.
G.
H.
I.
J.

Carotid stenosis
Rhabdomyolysis
Saphena varix
Intermittent claudication
Neuropathic ulcer
Spinal stenosis
Neuropathic ulcer
Saddle embolus
Deep venous thrombosis
Sturge-Weber syndrome

For each question below, choose the SINGLE most likely answer form the above list of
options. Each option may be used once, more than once, or not at all.

339. A 52-year-old patient with diabetes complains of an ulcer of his foot. He


has had diabetes for a long time and is erratic with taking medications.
There is a reduction in sensation to vibration and light touch in his foot.
340. A 74-year-old man presents with transient loss of vision. On examination
there is a bruit in the left carotid artery.
341. A 60-year-old man presents with a history of pain in the calf for the last 3
years. One year ago he was able to walk 300 yards but is now only able
to achieve 50 yards and has to rest between periods of walking.
342. A 22-year man presents to A&E following an accident at work. A piece of
scaffolding fell and trapped his left leg. His peripheral pulses are present
on arrival in hospital 2 hours after the accident. He is admitted for
observation when he becomes hypotensive with a poor urine output.
343. A 66-year-old man has undergone a major GI surgery and is recovering in
the hospital. He complains of pain in his right calf, which on examination
is tender, slightly swollen and red.

129

EMQs by Maju Mathews

Answers: Peripheral Vascular Disease


339. (E)
Commonly seen in patients with diabetes.

340. (A)
May cause TIAs Investigate with Duplex scan and later angiography.
Treatment is carotid endarterectomy.

341. (D)
Cramp like pain, which is brought on by walking, and relieved by rest.

342. (B)
Seen following trauma and destruction of muscle. Result in renal failure.

343. (H)
Seen in 30% of surgical patients. Venography is the definitive test.
Prevention is with early mobilization, heparin & support hosiery.

130

EMQs by Maju Mathews

Theme: Soft tissue swellings


Options
A.
B.
C.
D.
E.
F.
G.
H.
I.

Lipoma
Ganglion
Neuroma
Neurofibroma
Leiomyoma
Desmoid tumors
Kaposis sarcoma
Rhabdomyosarcoma
Bakers cyst

For each question below, choose the SINGLE most likely answer form the above list of
options. Each option may be used once, more than once, or not at all.

344. These lumps arise following trauma. The cells regenerate in a


disorganized manner. They are often found in amputation stumps
resulting in pain.
345. Rare tumors, which may be part of a genetic syndrome. Highly
vascularized fibrous tissue is present and they are not malignant.
Recurrence is common and most frequently found in the abdominal wall.
346. Arise in skeletal muscle. They most commonly affect children and can be
found in the head and neck, trunk and limbs.
347. Very common. Rarely become malignant. Usually asymptomatic but may
be multiple and painful.
348. Presents as purple macules on the skin and mucosa and tend to be
multiple. Seen in HIV positive patients.

131

EMQs by Maju Mathews

Answers: Soft Tissue Swellings


344. (C)
Regeneration of nerve endings.

345. (F)
AKA false neuromas consisting of fibrous tissue and coiled nerve fibbers.
Unencapsulated fibroma. Sometimes may occur repeatedly in spite of
adequate excision. Hard. Usually below the level of umbilicus.
Occasionally scars and other wounds. Intraperitoneal forms in Familial
Polyposis coli (Gardners syndrome). Treatment by wide excision

346. (H)
Tumour arising in muscle Head & neck sarcoma arises commonly in
pediatric age group.

347. (A)
Slowly growing tumor composed of fat cells. If causing trouble, removal is
indicated. If multiple and painful, neurolipomatosis. Decrums disease is an
associated condition.

348. (G)
Derived from capillary endothelial cells or fibrous tissue. It metastasizes to
lymph nodes. Diagnosis by biopsy. Treatment is by local radiotherapy,
interferon or chemotherapy.

132

EMQs by Maju Mathews

Theme: Male infertility


Options
A.
B.
C.
D.
E.
F.
G.
H.

Varicocele
History of Hodgkins Lymphoma
Peripheral vascular disease
Multiple sclerosis
Auto-antibodies
Retrograde ejaculation
Beta blocker medication
Depression

For each question below, choose the SINGLE most likely answer form the above list of
options. Each option may be used once, more than once, or not at all.

349. Erectile failure in a smoker of 40 cigarettes per day.


350. Normal sexual function but persistent azoospermic sample in a man who
suffers from recurrent urinary tract infections.
351. Oligospermia and scrotal swelling in a 24-year-old.
352. Erectile dysfunction in a 44-year-old hypertensive.
353. Azoospermia in a 58-year-old businessman 3 year after prostate surgery.

133

EMQs by Maju Mathews

Answers: Male Infertility


349. (C)
Peripheral vascular disease associated with buttock claudication and
impotence (Leriche Syndrome) is common in heavy smokers.

350. (F)
Retrograde ejaculation leads to azoospermia and may follow lower urinary
tract surgery or scarring.

351. (A)
Varicocele can usually be detected by palpating a soft scrotal swelling.

352. (G)
Drugs such as beta-blockers (and alcohol) may lead to erectile failure.

353. (F)
Is common following prostate surgery.

134

EMQs by Maju Mathews

Theme: Antibiotic prophylaxis of endocarditis


Options
A. Single dose 3-gm oral amoxicillin preprocedure
B. 1 gm i/v amoxicillin pre-procedure, then 500
mg oral amoxicillin
C. Single dose 600 mg oral clindamycin
D. 1 gm i/v amoxicillin + 120 mg gentamicin,
then 500 mg oral amoxicillin
E. 1 gm i/v vancomycin + 120 mg gentamicin
pre-procedure
F. None required
For each question below, choose the SINGLE most likely answer form the above list of
options. Each option may be used once, more than once, or not at all.

354. A 25-year-old lady with previous history of endocarditis undergoing


wisdom tooth extraction under general anesthesia.
355. A patient with atrial myxoma and penicillin-allergy undergoing dental
extraction under local anesthesia.
356. A 16-year-old boy known case of rheumatic heart disease under going a
tonsillectomy.
357. A patient who has a congenital ventricular septal defect undergoing dental
extraction under general anesthesia.
358. A patient with atrial septal defect undergoing dental extraction under
general anesthetic.
359. A pregnant woman with a Starr-Edwards mitral valve replacement
requiring a forceps delivery.

135

EMQs by Maju Mathews

Answers: Antibiotic Prophylaxis of Endocarditis


Page 138 OHCM
Local anesthetic + no risk
Single shot amoxicillin
If penicillin allergy give clindamycin
G/A (no risk)
Single shot amoxicillin i/v pre-procedure
Followed by another shot of amoxicillin.
G/A with endocarditis h/o prosthetic value
Pencilling + gentamicin pre-op followed by Penicillin Post-op
354.
355.
356.
357.
358.
359.

136

(D)
(C)
(A)
(B)
(F)
(D)

EMQs by Maju Mathews

Theme: Management of asthma in children


Options
A.
B.
C.
D.
E.
F.
G.
H.
I.
J.
K.

Inhaled low-dose corticosteroid


Inhaled high-dose corticosteroid
Oral steroid
Inhaled short-acting beta 2 stimulant
Inhaled long-acting beta 2 stimulant
Inhaled cromoglycate or oral necrodomil
Inhaled ipratropium
Oral beta 2 stimulant
Oral theophylline
Leukotriene antagonist
No treatment

For each question below, choose the SINGLE most likely answer form the above list of
options. Each option may be used once, more than once, or not at all.

360. A 14-year-old boy referred by his GP present with complaints of


occasional wheezing since 2 weeks. He is diagnosed to have mild
asthma. What treatment would you initiate?
361. He requires this drug more than once a day despite good compliance.
What do you add?
362. This patient continues to have nocturnal wheezing despite good
compliance with the above two medications. What would you substitute
the second drug with?
363. After a week, he reports back complaining of increased wheeze. He
continues to use the original medications properly and regularly. What do
you try now in place of the second drug?
364. He is still wheezy. What do you try in addition to the current two drugs?
365. He developed side effects with the last drug, what are your going to try
now?

137

EMQs by Maju Mathews

Answers: Management of Asthma in Children


360.
361.
362.
363.
364.
365.

(D)
(F)
(A)
(B)
(E)
(I)

Note: The following sequence is followed in the treatment of bronchial asthma


in children (BTS guidelines). (OHCS 270)
Step-I
Step II
Step III
Step IV
Step V

138

Occasional beta agonists. If needed > daily.


Cromoglycate an inhaled powder
Swap cromoglycate for budesonide 50-200 mcgm/12-hr by spacer
Increase budesonide to 400-mcgm/1-hrs + prednisolone +/- long
acting beta agonist
Slow release xanthine +/- nebulized beta agonist +/- alternate day
prednisolone + - ipratropium or beta agonist infusion

EMQs by Maju Mathews

Theme: Causes of Nystagmus


Options
A.
B.
C.
D.
E.
F.
G.
H.
I.
J.
K.
L.

Physiological
Vestibular neuronitis
Mnires disease
Benign positional vertigo
Multiple sclerosis
Vertebrobasilar ischaemia
Wernickes encephalopathy
Phenytoin toxicity
Left cerebellar infarction
Right cerebellar infarction
Korsakoffs syndrome
Vestibular schwannoma

For each question below, choose the SINGLE most likely answer form the above list of
options. Each option may be used once, more than once, or not at all.

366. A 45-year-old alcoholic presents with vomiting and falls. On examination,


GCS is 10 with nystagmus in all directions of gaze and an ataxic gait.
367. A 50-year-old woman presents with intermittent vertigo, tinnitus and
deafness in the right ear. On examination she has horizontal nystagmus
on looking to the right.
368. A 30-year-old presents with intermittent giddiness. On examination, three
beats of nystagmus at the extremes of left and right gaze.
369. A 60-year-old man brought by his wife who reports increasing clumsiness
while performing simple daily tasks and intermittent giddiness. O/E right
sided tremor with ataxic gait and nystagmus on looking to the right.
370. A 20-year-old presents with acute onset of vertigo and nausea. He vomits
on attempting to stand erect. On examination he has nystagmus on left
gaze.
371. 25-year-old male was involved in an RTA 2-weeks back. He now has
vertigo worsened by any movement.
372. A 55-year-old male presents with hearing loss in the right ear, nystagmus
on right gaze with absent corneal reflex in right eye.

139

EMQs by Maju Mathews

Answers: Causes of Nystagmus


366. (G)
Thiamine deficiency state: triad of nystagmus, ophthalmoplegia and
ataxia. Ptosis, abnormal papillary reactions and altered consciousness.
Headache, anorexia, vomiting. Treatment: Thiamine

367. (C)
Above symptoms in clusters, vomiting. Symptoms lasting 12-hours.
Deafness sensorineural. Progressive
Symptomatic treatment with cyclizine/betahistine
Operative decompression of saccus endolymphaticus best treatment.

368. (A)
Pathological nystagmus >3 times.

369. (I)
Cerebral tr___, ataxia, asthenia, atonia.

370. (B)
Vomiting, vertigo and prostration worsened by movement. Cyclizine.

371. (D)
Post head injury, spontaneous degeneration of labyrinth, post viral illness,
stapes surgery, chronic middle ear disease.

372. (L)
Involves the trigeminal facial nerve vestibules cochleas nerves.

140

EMQs by Maju Mathews

Theme: Causes of weak legs


Options
A.
B.
C.
D.
E.
F.
G.
H.
I.
J.
K.
L.

Spinal cord compression


Myelitis
Friedreichs ataxia
Multiple sclerosis
Cauda equina compression
Guillain-Barre syndrome
Hypokalemia
Syringomyelia
Motor neurone disease
B12 deficiency
Peripheral neuropathy
Myasthenia gravis

For each question below, choose the SINGLE most likely answer form the above list of
options. Each option may be used once, more than once, or not at all.

373. A 25-year-old woman presents with increasing bilateral leg weakness.


O/E, she has increased tone and exaggerated reflexes in both legs with
extensor plantar response. She also has pale optic discs on fundoscopy.
374. A 60-year-old man presents with progressive leg weakness. On
examination, he has wasting of muscles of lower limb with fasciculation
and power 2/5. Knee and ankle jerks are brisk and he has ankle clonus.
375. A 25-year-old man presents with a short history of progressive weakness
of his legs with some weakness of the arms also. On examination, he has
a flaccid weakness of his limbs with absent reflexes. FVC is reduced.
376. A 55-year-old woman presents with falls and weak legs for three years.
On examination, she has pes cavus and wasting of the lower legs. She
has a wide-based ataxic gait and Rombergs test is positive. Ankle jerks
are absent and vibration sense is lost to the knees. Plantars are down
going.
377. A 50-year-old woman presents with weakness of hands and legs since a
year. On examination loss of pain and temperature sensation over arms
and trunk. Knee and ankle jerks are brisk with increased tone in lower
limbs and extensor plantars.
378. A 40-year-old man with a recent history of back pain after lifting a crate
presents with a 24-hour history of bilateral leg weakness and difficulty
passing urine. He has increased leg tone and brisk knee and ankle jerks.

141

EMQs by Maju Mathews

Answers: Causes of Weak Legs


373. (D)
Young adults. Episodic demyelination throughout the CNS. Peripheral
nerves unaffected. Usual presentation with optic neuritis, isolated
numbness, or with weakness of legs. Other features: vertigo, nystagmus,
double vision, pain, incontinence, cerebellar signs, Lhermittes sign, facial
palsy, epilepsy, aphasia, euphoria, dementia.
374. (I)
Both upper & lower motor neurons affected. Never any sensory
abnormality, Never affects extraocular muscles. This part has Amyotrophic
lateral sclerosis (combined LMN wasting & UMN hyperreflexia). Other
types: Progressive muscular atrophy (Anterior horn cells, Distal muscles
before proximal) and Bulbar palsy.
375. (F)
Acute post infective polyneuritis. Few weeks following mild infections,
motor polyneuropathy. Ascending paralysis. Proximal muscles more
affected. Trunk, resp and cranial nerves. Sensory involvement common.
Supportive treatment. Recover in 3-6 months.
376. (K)
Rombergs test is positive in loss of joint position sense, which in
combination with the other signs is found in peripheral neuropathy.
377. (H)
Presents with weakness of the limbs with wasting of the arms & hand with
loss of pain & temp over the trunk and arms (dissociated sensory loss)
cape distribution. UMN signs in lower limbs. Horners syndrome, Charcot
joints.
378. (A)
Spinal cord compression is an emergency & needs urgent treatment.
Involvement of the sphincters is an ominins sign.

142

EMQs by Maju Mathews

Them: Diagnosis of Earache


Options
A.
B.
C.
D.
E.
F.
G.
H.
I.
J.

Aerotitis
Otosclerosis
Otitis Media
Cholesteatoma
Glue ear
Furunculosis
Chronic suppurative otitis media
TM joint dysfunction
Bullous myringitis
Mastoiditis

For each question below, choose the SINGLE most likely answer form the above list of
options. Each option may be used once, more than once, or not at all.

379. A 34-year-old man with seborrheic dermatitis develops pain and discharge
from the right ear. Movement of the auricle causes severe pain.
380. A 66-year-old IDDM develops severe ear pain on jaw movement. Tender
tragus and boil seen on the external auditory meatus.
381. A 41-year-old woman develops severe left ear pain, foul smelling
discharge associated with sensorineural deafness. O/E left sided facial
palsy and attic perforation seen.
382. A 14-year-old girl complains of sudden onset left ear pain and deafness
on a intercontinental flight. O/E fluid level and hemorrhagic areas seen on
the drum.
383. An 8-year-old child develops fever, left sided ear pain, reduced hearing
and foul smelling discharge. O/E downward displacement of the pinna
noticed.

143

EMQs by Maju Mathews

Answers: Diagnosis of Earache


379. (C)
Is a common association

380. (F)
Staphylococcal infection is common in patients with IDDM.

381. (D)
The stratified squamous epithelium erodes into various structures.

382. (A)
Glue ear can predispose to aerotitis.

383. (J)
Recognized complication of untreated otitis media.

144

EMQs by Maju Mathews

Theme: Investigation in overdose


Options
A.
B.
C.
D.
E.
F.
G.
H.
I.
J.
K.
L.

U&E
Prothrombin time
Serum Calcium
APTT
ECG
Chest x-ray
ABGs
FBC
Serum Lithium
LFT
TFT
Platelet count

Select the MOST appropriate investigation for each of the clinical scenarios listed
below. Each choice may be used once, more than once, or not at all.

384. A 33-year-old female was brought to the hospital in a drowsy state


following an overdose of Dothiepin.
385. A 64-year-old man on prophylaxis for DVT was brought to A&E after
having consumed a large quantity of his medication.
386. An 18-year-old adolescent has taken about 50 aspirin tables. He is
brought to A&E with sweating, vomiting and dehydration.
387. A 34-year-old man was brought to A&E having consumed excess tablets
of Salbutamol with vomiting, tremors and palpitations.
388. A 35-year-old female with Bipolar Affective Disorder is brought in a
confused state having taken an overdose of her medications.

145

EMQs by Maju Mathews

Answers: Investigation in Overdose


384. (E)
Tricyclics have adverse effects on the heart and hence an ECG should be
performed in addition to the other investigations.

385. (B)
The overdose is likely to be Warfarin.

386. (G)
Comes metabolic acidosis.

387. (A)
Risk of hypokalemia.

388. (I)
The most likely medication that he is likely to have taken is Lithium.

146

EMQs by Maju Mathews

Theme: Diagnosis of bacterial infections


Options
A.
B.
C.
D.
E.
F.
G.
H.
I.
J.
K.
L.

Streptococcus pneumoniae
Clostridium botulinum
E. coli
Tuberculosis
Leptospirae
Gonorrhoeae
Salmonella typhi
Borrelia burgdorferi
C. difficile
Staph aureus
Meningitidis
Legionella pneumophilia

Select the MOST appropriate investigation for each of the clinical scenarios listed
below. Each choice may be used once, more than once, or not at all.

389. A 17-year-old girl with sickle cell disease developed gradual onset right
knee joint pain and swelling associated with fever and inability to move
the limb.
390. A 48-year-old farmer develops fever, headache, malaise and myalgia. He
is a water sport enthusiast and had taken part in water sports in inland
waters a week prior to this.
391. A 73-year-old man develops bloody diarrhea and abdominal cramps. He
was hospitalized 10 days back for an infected wound on the leg.
392. A 45-year-old executive with fever and cough, he recently returned from a
conference abroad.
393. A 13-year-old boy develops sudden onset generalized fatigue, abdominal
discomfort, blurred vision and diplopia. History of eating hazelnut yoghurt
the previous night.

147

EMQs by Maju Mathews

Answers: Diagnosis of Bacterial Infections


389. (G)
S. typhi is the commonest organism causing Osteomyelitis in patients with
sickle cell disease.
.
390. (E)
Affects people involved in sewage treatment plants or those who involve
in inland waters.

391. (I)
Pseudomembranous colitis is seen in patients treated on broad spectrum
antibiotics.

392. (L)
This is commonly transmitted through contaminated air conditioning
systems.

393. (B)
This is one mode of transmission.

148

EMQs by Maju Mathews

Theme: Diagnosis of abdominal pain


Options
A.
B.
C.
D.
E.
F.
G.
H.
I.

Bowel perforation
Gastric ulcer
Duodenal ulcer
Acute cholecystitis
Oesophageal varices
Splenic rupture
Fracture femur
Bulimia nervosa
Anorexia nervosa

Select the MOST appropriate investigation for each of the clinical scenarios listed
below. Each choice may be used once, more than once, or not at all.

394. A 39-year-old female presents with fever, severe abdominal pain and
tenderness. Her chest x-ray reveals free air under the diaphragm.
395. A 40-year-old lady, known smoker presents with mid epigastric pain
relieved by ingestion of milk.
396. A 45 -year-old obese female presents with high fever, vomiting and severe
right upper quadrant pain. Tenderness is elicited on palpation at the
midpoint of the right subcostal margin on inspiration.
397. A 35-year-old man is involved in an RTA. He presents a few hours later to
A&E with a bloated abdomen & pain in the left shoulder.
398. A 30-year-old lady present with severe abdominal pain. Examination
revealed anaemia, enlarged parotid glands, callouses on the knuckles
and dental erosions.

149

EMQs by Maju Mathews

Answers: Diagnosis of Abdominal Pain


394. (A)
Free air under the diaphragm is a classical finding in bowel perforation.

395. (C)
This is a typical presentation.

396. (D)
Common organisms are E. coli, Klebsiella, and Streptococcus faecalis.

397. (F)
Kehrs sign, due to blood in contact with the undersurface of the
diaphragm.

398. (H)
One to induced vomiting. Can cause rupture of the stomach.

150

EMQs by Maju Mathews

Theme: Brain region localization


Options
A.
B.
C.
D.
E.
F.
G.
H.
I.

Nondominant frontal lobe


Dominant frontal lobe
Nondominant temporal lobe
Dominant parietal lobe
Nondominant parietal lobe
Dominant temporal lobe
Occipital lobe
Temporo-occipito parietal junction
Cerebellum

Select the MOST appropriate investigation for each of the clinical scenarios listed
below. Each choice may be used once, more than once, or not at all.

399. A 67-year-old stroke patient cannot tell his left hand from his right hand.
400. A 50-year-old man with stroke can understand what others say to him but
is unable to respond verbally.
401. A patient with stroke cannot add a column of 4 single digit numbers.
402. A 55-year-old stroke patient cannot copy a simple drawing.
403. A 65-year-old man presents with a history of falling to one side. On
examination, he has dysarthria, hypotonia and nystagmus.

151

EMQs by Maju Mathews

Answers: Brain Region Localization


399.
400.
401.
402.
403.

152

(D)
(B)
(D)
(E)
(I)

EMQs by Maju Mathews

Theme: Cardiotocography (CTG)


Options
A.
B.
C.
D.
E.
F.
G.

Foetal anaemia
Foetal cardiac arrhythmia
Foetal heart block
Beta sympathomimetic drug use
Foetal hypoxia
Raising foetal intracranial pressure
Cord compression

For each question below, choose the SINGLE most likely answer form the above list of
options. Each option may be used once, more than once, or not at all.

404. A 30-year-old female at 37 weeks gestation. The pregnancy has been


uneventful so far. A routine CTG showed a loss of base line variability.
405. CTG shows variable decelerations.
406. CTG shows late decelerations.
407. CTG shows baseline tachycardia of >200. Mother is afebrile. She has not
been taking any medications.
408. CTG shows spasmodic bradycardia.

153

EMQs by Maju Mathews

Answers: Cardiotocography (CTG)


404. (E)
May also suggest sleeping foetus or drug effects.

405. (G)

406. (E)
Suggest Foetal hypoxia.

407. (B)

408. (G)

154

EMQs by Maju Mathews

Theme: Prophylactic treatment


Options
A.
B.
C.
D.
E.
F.
G.
H.
I.
J.
K.
L.

Vancomycin and gentamicin


Cefuroxime and gentamicin
Ampicillin IV
Sodium stibogluconate
Mefloquine
Chloroquine and proguanil
Zidovudine
Rifampicin
Isoniazid
Tetracycline
Clindamycin
Interferon

For each question below, choose the SINGLE most likely answer form the above list of
options. Each option may be used once, more than once, or not at all.

409. A 45-year-old lady due to undergo cholecystectomy at the city hospital.


410. An 18-year-old girl, known case of rheumatic heart disease due for dental
scaling under local anerethisia. She is allergic to penicillin.
411. A mother brings her 4-year-old child to A&E worried that she may have
been exposed to meningococcal meningitis following and outbreak in the
local nursery.
412. A 30-year-old pregnant lady going on vacation to Sub-Saharan Africa for
one month. She is worried about contracting malaria and would like
adequate protection to prevent infection.
413. A 25-year-old pregnant HIV positive woman concerned about minimizing
the risk of transmission of HIV to her fetus.

155

EMQs by Maju Mathews

Answers: Prophylactic Treatment


409. (C)
For biliary surgery ampicillin 500-mg iv/8-hr for 3 doses or cefuroxime
750 mg/ 8h for 3 doses iv / im.
410. (K)
Clindamycin 600-mg 1-hour before local anesthetic procedure. If can
tolerate penicillin, amoxicillin 3 gram 1 hour before procedure.
411. (H)
Rifampin is indicated in the treatment of closed contacts of patients
with proved or suspected infection caused by Neisseria meningitides.
These contacts include other household members, children in
nurseries, persons in day care centers, and closed populations, such
as military recruits. Health care providers who have intimate exposure
(e.g. mouth-to-mouth resuscitations) with index cases also should
receive prophylactic therapy. Household members and other close
contacts of patients with recently diagnosed tuberculosis who have a
positive tuberculin skin test (PPD) of 5-mm, [tuberculin-negative
children and adolescents who have been close contacts of infections
person within the past 3 months are also candidates for preventative
therapy until a repeat PPD is done 12 weeks after contact with the
infectious source].
412. (F)
Chloroquine crosses the placenta. Use is not recommended during
pregnancy except in the suppression or treatment of malaria or hepatic
amebiasis since malaria poses greater potential danger to the mother
and fetus (ie. abortion and death) than prophylactic administration of
chloroquine. Chloroquine, given in weekly chemoprophylactic doses,
has not been shown to cause adverse effects on the fetus.
413. (G)
Zidovudine is indicated for the prevention of maternal-Foetal HIV
transmission as part of a regiment that includes oral zidovudine
beginning between 14 and 34 weeks gestation, continuous intravenous
infusion of zidovudine during labor, and administration of zidovudine
syrup to the neonate for the first six weeks of life. However,
transmission to infect may still occur in some cases despite the use of
this regimen.

156

EMQs by Maju Mathews

Theme: Diagnosis of poisoning


Options
A.
B.
C.
D.
E.
F.
G.
H.
I.
J.
K.
L.
M.

Benzodiazepines
Tricyclic antidepressants
Carbon monoxide
Organophosphate insecticides
Lithium
Carbon tetrachloride
Ethylene glycol
Paracetamol
Cyanide
Strychnine
Lead poisoning
Antifreeze
Paraquat

For each question below, choose the SINGLE most likely answer form the above list of
options. Each option may be used once, more than once, or not at all.

414. A 23-year-old girl presents with coma and convulsions. One examination
tachycardia, hyperreflexia and dilated pupils.
415. A young couple brought to A&E with acute onset of fever, headache,
restlessness and nausea. On examination GCS is 9 and the skin shows a
pink color.
416. A 6-year-old child brought with burns in mouth, dysphagia, abdominal pain
and distension. There is progressive respiratory distress with hypoxemia.
417. A 6-year-old child presents with restlessness, pyrexia and convulsions. On
examination, hypertonia and hyperflexia.
418. A 35-year-old female brought to A&E in a comatose state. She is noticed
to have flushed pink skin and her breath has a distinctive odor of bitter
almonds.
419. A 27-year-old industry worker presents with sudden onset of vomiting,
diarrhea, abdominal pain and seizures following accidental ingestion of
solvent. On examination tender hepatomegaly and jaundice.
420. A 30-year-old woman with known history of psychiatric illness, on
medication, brought with tremors and convulsions. On examination
hyperflexia, rigidity and nystagmus. Investigations shows hypokalemia.

157

EMQs by Maju Mathews

Answers: Diagnosis of Poisoning


414. (B)
TCA overdose. Phenothiazine also presents similarly.

415. (C)
Probably by falling asleep in a closed vehicle with engine left running.

416. (M)
Accidental ingestion by children common.

417. (J)
Can mimic tetanus.

418. (I)
May follow excessive use of sodium nitroprusside. Treated with dicobate
edetate or sodium nitrite.

419. (F)
Used as a solvent in industrial processes. Rapid onset of symptoms.

420. (E)
On lithium for bipolar disorder with acute overdose.

158

EMQs by Maju Mathews

Theme: Diagnosis of pruritus


Options
A.
B.
C.
D.
E.
F.
G.
H.
I.
J.
K.
L.

Primary biliary cirrhosis


Psoriasis
Dermatitis herpetiformis
Urticaria
Hypothyroidism
Lymphoma
Atopic eczema
Seborrheic dermatitis
Scabies
Lichen planus
Uremia
Polycythemia

For each question below, choose the SINGLE most likely answer form the above list of
options. Each option may be used once, more than once, or not at all.

421. A 35-year-old male present with and itchy scaly rash particularly over his
wrists. Rash is papular and mauve topped. His buccal mucosa shows lace
like steaks.
422. A 30-year-old nurse presents with generalized itching. There are
widespread excoriation marks on her trunk and arms, with linear tracts
around his wrists and between his fingers.
423. A 25-year-old woman complains of itching since 2 days. There is whealing
of the skin after she has scratched it, which lasts for up to an hour.
424. A 50-year-old woman complains of pruritus and skin pigmentation. She is
found to have anti-mitochondrial antibodies.
425. A 65-year-old man complains of malaise, lethargy and pruritus. He also
has nocturia and polyuria, with nausea and vomiting. On examination
pallor, pigmentation with sings of fluid overload.

159

EMQs by Maju Mathews

Answers: Diagnosis of Pruritus


421. (J)
Lichen planus (LP) is a papulosquamous disorder in which the primary
lesions are pruritic, polygonal, flat-topped, violaceous papules. Close
examination of the surface of these papules will often reveal a network of
gray lines (Wickhams striae). The skin lesions may occur anywhere, but
they have a predilection for the wrists, shins, lower back, and genitalia.
Involvement of the scalp may lead to hair loss. LP commonly involves
mucous membranes, particularly the buccal mucosa, where it can present as
a white netlike eruption. Its etiology is unknown.
422. (I)
Transfer of newly fertilized female mites from person to person occurs by
intimate personal contact and is facilitated by crowding, uncleanliness, and
sexual promiscuity. Medical practitioners are at particular risk of infestation.
Outbreaks occur in nursing homes, mental institutions, and hospitals.
Patients with scabies report intense itching that worsens at night and after a
hot shower. Typical burrows may be difficult to find because they are few in
numbers and may be obscured by excoriations. The possibility of other
sexually transmitted diseases should be excluded in adults with scabies.
Sites include finger webs, wrists, elbows, ankles, genitalia and breasts. Treat
with Malathion or permethrin.
423. (D)
Urticaria (hives) are transient lesions that are composed of a central wheal
surrounded by an erythematous halo. Individual lesions are round, oval or
figurate, and they are often pruritic. Acute and chronic urticaria has a wide
variety of allergic etiologies.
424. (A)
Antimitochondrial antibody (AMA) is detected in more than 90 percent of
patients with primary biliary cirrhosis amount patients with symptomatic
disease, 90 percent are women aged 35 to 60. Often the earliest symptom is
pruritus, which may be either generalized or limited initially to the palms and
soles. In addition, fatigue is commonly a prominent early symptom. After
several months or years, jaundice and gradual darkening of the exposed
areas of the skin (melanosis) may ensue.
425. (K)
Uremic symptoms include fatigue, breathlessness, ankle swelling, anorexia,
vomiting, nocturia and pruritus. Examination can reveal pigmentation, pallor
and brown nails.

160

EMQs by Maju Mathews

Theme: Diagnosis of sore throat


Options
A.
B.
C.
D.
E.
F.
G.
H.

Acute otitis media


HIV
Infectious mononucleosis
Diphtheria
Agranulocytosis
Scarlet fever
Acute follicular tonsillitis
Pharyngitis

Instructions: Match each clinical scenario below with the MOST appropriate diagnosis
from the list of the options above. Each option may be used once, more than once and
not at all.

426. A 30-year-old woman presents to her GP with score throat. She is being
treated for thyrotoxicosis currently.
427.A 7-year-old child is presented to his GP with sore throat, a punctate
erythematous rash and circumoral pallor.
428. A 4-year-old child complain of sore throat and earache. He is pyrexial. On
examination tonsils enlarged and hyperemic exude pus when pressed
upon.
429. A 30-year-old man presents in A&E department with difficulty in breathing.
He has returned from India recently. Examination of his throat revealed
grey membranes on the tonsils and uvula. Pyrexia is mild.
430. A 23-year-old man presents with severe malaise and sore throat. He has
red tonsils with yellow membranes and cervical lymphadenopathy.

161

EMQs by Maju Mathews

Answers: Diagnosis of Sore Throat


426. (E)
Agranulocytosis is a recognized side effect of carbimazole, used for the
treatment of thyrotoxicosis. Patients receiving this drug should be warned,
therefore, to see their GP if sore throat/mouth ulcers/fever develops.
427. (F)
It is difficult to make a mistake dealing with such a bright clinical picture.
In addition these patients have a characteristic tongue, which is described
as strawberry and cream.
428. (G)
The child has typical follicular tonsillitis. The pus exuding from the crypts
is characteristic. Earache in that case is usually referred.
429. (D)
Diphtheria has become rare in the UK but it is emerging again in some
parts of eastern Europe and south east Asia. Hence, recent travel to these
continents is characteristic. Diphtheria presents sometimes as an
emergency when severe oedema and membranes lead to airway
obstruction and as phyxia. Firmly attached grey membranes that are
difficult to remove and that leave a bleeding surface after an attempt to
peel them off point towards the diagnosis.
430. (C)
The clues to the diagnosis in this young man are malaise, which is often
more severe than in acute tonsillitis, yellow membranes on the tonsils and
enlarged lymph nodes. Diagnosis is confirmed by lymphocytosis in the
peripheral blood and positive Paul-Bunnel test.

162

EMQs by Maju Mathews

Theme: Treatment of Diabetic Complication


Options
A.
B.
C.
D.
E.
F.
G.
H.
I.
J.
K.
L.

Oxybutynin
Amitriptyline
Atenolol
Captopril
Dexamethasone
Fludrocortisone
Metoclopramide
Metronidazole
Ampicillin
Tetracycline
Testosterone
Sildenafil

For each question below, choose the SINGLE most likely answer form the above list of
options. Each option may be used once, more than once, or not at all.

431. A 60-year-old type-2 diabetic female complains of dizziness following


exercise or after a heavy meal. She is noted to have a blood pressure of
120/80 in supine position and 90/65-mmHg in the erect position.
432. A 40-year-old IDDM female complains of weight loss. Breath hydrogen
analysis shows increased exhaled hydrogen and reduced gut motility.
433. A 43-year-old male diabetic complains of dysphagia and vomiting. Barium
swallow done shows a gastric dilatation and reduced motility.
434. A 63-year-old type-2 female diabetic complains of incontinence.
Investigation reveals a sphincter dyssynergia.
435. A 60-year-old type-2 diabetic male complains of decreased libido and
importance. The only abnormalities confirm a primary hypogonadism.
436. A 45-year-old male diabetic is admitted with evidence of peripheral
vascular disease. He is found to have hypertension.

163

EMQs by Maju Mathews

Answers: Treatment of Diabetic Complications


431. (F)
Fludrocortisone is used in conjunction with increased sodium intake in the
treatment of idiopathic orthostatic hypotension.
432. (I)
This patient has bacterial overgrowth as illustrated by the positive
hydrogen breath test. The most suitable antibiotic to reduce overgrowth
would be tetracycline.
433. (G)
Metoclopramide is indicated for the relief of symptoms of acute and
recurrent diabetic gastroparesis. Previously, cisapride was a particularly
good treatment although this has been withdrawn due to the prolongation
of the QT-interval.
434. (A)
Oxybutynin is indicated for the relief of symptoms associated with voiding,
such as frequent urination, urgency, urge incontinence, nocturia, and
incontinence in those patients with reflex neurogenic bladder.
435. (K)
This patient has primary hypogonadism, which is a reduced testosterone
and raised LH/FSH. This most appropriate therapy is testosterone
replacement.
436. (D)
ACE inhibitors are used for treatment of essential, malignant, refractory,
or accelerated hypertension, and for treatment of renovascular
hypertension (except in patients with bilateral renal artery stenoses or
renal artery stenosis in a solitary kidney). Captopril has been shown to
slow the progression of diabetic nephropathy in normotensive and
hypertensive IDDM patients with documented diabetic retinopathy.

164

EMQs by Maju Mathews

Theme: Chest pain


Options
A.
B.
C.
D.
E.
F.
G.
H.
I.
J.
K.

Angina
Tietzes syndrome
Aortic dissection
Tabes dorsalis
Esophagitis
Pleuritic pain
Neurosis
Pericarditis
Myocardial infarction
Da Costas syndrome
Chronic pain syndrome

For each question below, choose the SINGLE most likely answer form the above list of
options. Each option may be used once, more than once, or not at all.

437. A 55-year-old male presents to A&E with intense dull central chest pain
with associated nausea and vomiting lasting 3-hours. On examination
pulse is irregular and BP is 96/60-mmHg.
438. A 50-year-old lady presents with history of cheat pain of continually
varying nature. Investigations done suggest no apparent cause. She is
prescribed a course of low dose tricyclics with which she responds
favorably.
439. A 40-year-old man reports with a constricting epigastric pain, which
occasionally radiates to the neck and jaw, precipitated following his meals
and increases in the supine position. The pain improves slowly with the
administration of glyceryl trinitrate.
440. A 55-year-old lady presents with a sharp constant sternal pain, which is
worse on inspiration, coughing and is relieved on sitting forward. She was
admitted with a diagnosis of MI ten days back. On examination, superficial
sound heard best at left sternal edge.
441. A 22-year-old male presents with a sharp localized chest pain exacerbated
on running or sneezing. He is worried that it may be cardiac in origin. O/E,
marked tenderness over the sternal edge of 2nd rib.

165

EMQs by Maju Mathews

Answers: Chest pain


437. (I)
Wide range of symptoms.

438. (K)
Recognized syndrome & no apparent cause but respond to low dose
tricyclics.

439. (E)
Pain radiating to the back worsening with steeping and has _______. Can
result in anaemia, strictures and Barretts ulcer.

440. (H)
Sharp constant sternal pain relieved by sitting forward ______________
left shoulder. May be made worse by lying on ____________________ &
swallowing. Also Known as __________ syndrome. Treat with _____

441. (B)
Costochondritis. Treat with NSAIDs.

166

EMQs by Maju Mathews

Theme: Diagnosis of falls


Options
A.
B.
C.
D.
E.
F.
G.
H.
I.
J.
K.
L.

Stokes Adams attack


Postural hypotension
Sick sinus syndrome
Vasovagal episode
Acute labyrinthitis
Benign positional vertigo
Menires disease
Parkinsons disease
Sensory ataxia
Cerebellar involvement
Impaired vision
Drop attacks

For each question below, choose the SINGLE most likely answer form the above list of
options. Each option may be used once, more than once, or not at all.

442. A 75-year-old woman presents with history of several falls over the past 5
months. She says that they come on suddenly without warning and
reports no loss of consciousness.
443. A 70-year-old woman brought by her daughter with a history of being off
legs. Her balance has been poor for some time and she walks with a
stamping gait. Rombergs test is positive.
444. A 65-year-old man presents with history of several falls over the past one
year. He finds it increasingly difficult to dress by himself and carry out
activities of daily living. Examination revealed a tremor in the upper limb
with hypotonia. He could not stand erect even with his eyes open.
445. A 60-year-old lady complains of giddiness and vomiting since the last 3
days. Her symptoms are exacerbated by head movements and she
cannot stand without support.
446. A 58-year-old woman presents with recurrent falls associated with
giddiness, particularly on getting up in the mornings. She has underlying
ischaemic heart disease and is on aspirin, an oral nitrate and a betablocker.

167

EMQs by Maju Mathews

Answers: Diagnosis of falls


442. (L)
Drop attacks resulting from impaired blood supply to the motor pathways
of the brainstem cause sudden paraparesis or quadriparesis, usually
lasting only a few seconds. Sudden drop attacks may mimic syncope.
Isolated loss of consciousness is rare.
443. (I)
Imbalance with sensory ataxia is characterized by marked worsening
when visual feedback is removed. The patient can often assume the
upright stance with feet together cautiously with eyes open. With eye
closure, balance is rapidly lost (positive Rombergs signs) in various
directions at random. Sensory examination reveals impairment of
proprioception at the toes and ankles, usually associated with an even
more prominent abnormality of vibratory perception. Prompt evaluation for
vitamin B12 deficiency is important, as this disorder is reversible if
recognized early. Depression or absence of reflexes points to peripheral
nerve disorders. Spasticity with extensor plantar responses suggest
posterior column and spinal cord disorders.
444. (J)
Once a desired position is reached, imbalance may be surprisingly mild.
As walking begins, the imbalance recurs. Patients usually learn to lessen
the imbalance by walking with the legs widely separated. Clinical signs of
cerebellar
limb
ataxia
include
dysmetria,
intention
tremor,
dysdiadochokinesia, and abnormal rebound. Muscle tone is often
modestly reduced; this contributes to the abnormal rebound due to
decreased activation of segmental spinal cord reflexes and also to
pendular reflexes, i.e, a tendency for a tendon reflex to produce multiple
swings to and fro after a single tap.
445. (E)
Imbalance with vestibular dysfunction is characterized by a consistent
tendency to fall to one side. The patient commonly complains of vertigo
rather than imbalance, especially if the onset is acute. Acute vertigo
associated with lateralized imbalance but no other neurologic signs is
often due to disorders of the semicircular canal.
446. (B)
Hypotension is a side-effect of most anti-anginal medications.

168

EMQs by Maju Mathews

Theme: Diagnosis of constipation


Options
A.
B.
C.
D.
E.
F.
G.
H.
I.
J.
K.

Dietary Deficiency
Old age
Diverticulosis
Crohns disease
Medication related
Autonomic neuropathy
Irritable bowel syndrome
Spinal injury
Multiple sclerosis
Colorectal carcinoma
Fissure in ano

447. A 40-year-old female presents with abdominal pain and distension relieved
by passage of flatus. She also complains of dyspareunia, urinary
frequency and altered bowel habits. She says that she has suffered
numerous such episodes in the past 5 years, which remit spontaneously.
Examination is unremarkable.
448. A 65-year-old female complains of increasing constipation since 5 days.
She is in hospital for a week following a fall sustaining a fracture neck of
femur.
449. A 50-year-old male presents with a history of abdominal pain and altered
bowel habits. He also reports a weight loss of 20 lbs over the past 3
months. Examination shows pallor and a mass in the left iliac fossa.
450. A 30-year-old female presents with episodes of numbness and tingling
over his hands and difficulty in walking. The episodes spontaneously
subside with full recovery. She has also developed recent onset
constipation and urinary incontinence.
451. A 28-year-old male homosexual presents with passage of hard stools and
painful defaecation since one week. He is uncomfortable in the sitting
position often moving to the edge of the chair. Examination shows the
presence of a sentinel pile.

169

EMQs by Maju Mathews

Answers: Diagnosis of constipation


447. (G)
International diarrhea, abdominal colic relieved by bowel action and
bloated feelings. Diagnosis of exclusion.

448. (E)
Lack of mobility can cause constipation.

449. (J)
Presentation depends on the side. Right side: weight loss, abdominal pain
left side: Bleeding PR, altered bowel habits, mass PR and tenesmus.

450. (I)
Myriad symptom complex with multisystem involvement

451. (K)
Midline longitudinal split in the squamous lining of the lower anus.

170

EMQs by Maju Mathews

Theme: Treatment of urinary incontinence


Options
A.
B.
C.
D.
E.
F.
G.
H.
I.
J.
K.

Oxybutynin
Anti-cholinergics
Ventricular shunting
Prostate surgery
Radiotherapy
Supportive care
Pelvic floor exercises
Bladder catheterization
Spinal surgery
Bladder retraining
Vaginal pessary

For each question below, choose the SINGLE most likely answer form the above list of
options. Each option may be used once, more than once, or not at all.

452. A 70-year-old male with known prostatic carcinoma presents with an 8hour history of being off legs and incontinence. Examination reveals
spastic paraparesis with a sensory impairment at L1.
453. A 52-year-old woman presents with urinary incontinence on coughing or
sneezing, increasingly becoming worse following the birth of her second
child.
454. A 77-year-old man is admitted with two-month history of cognitive
impairment, off legs and urinary incontinence. Examination reveals a gait
apraxia.
455. A 40-year-old female with multiple sclerosis presents with urinary
incontinence without urgency or frequency, worse over the last 6 months.
She has evidence of spastic paraparesis with cerebellar syndrome.
456. A 65-year-old woman complains of urinary frequency and urgency. She
does not like taking tablets. Examination is normal.

171

EMQs by Maju Mathews

Answers: Treatment of urinary incontinence


452. (E)
The history is suggestive of acute cord compression, which can cause
overflow incontinence, MRI is the investigation of choice. Radiotherapy is
usually the treatment of choice, especially when metastatic disease is
present. For localized cord compression from a newly diagnosed tumour
or tumours with poor response to radiotherapy e.g. adenocarcinoma of
lung or bowel, decompression surgery is alternative.
453. (G)
Post-natal pelvic floor muscle exercises are useful in preventing stress
inconti-nence. The right movement is when urine is stopped mid-stream.
454. (C)
The classical presentation of normal pressure hydrocephalus is dementia,
gait disturbance (pathognomonic glued-to-the-floor sing) and
incontinence. CT head shows disproportionate ventricular englargement
compared with degree of cortical atrophy. Ventricular shunting is the
definitive treatment.
455. (H)
Failure to fill and empty (uncoupling of detrusor and internal sphincter
function) occurs with detrusor contractions against a closed sphincter.
There will be overflow incontinence for which intermittent selfcatheterization is the treatment of choice. Anticholinergics can be tried if
urgency/frequency present.
456. (J)
Bladder retraining exercises is useful in urge incontinence. Increase
intervals of regular toileting from initial 30 minutes, increasing by 10-15
minutes every few days until intervals of 2-4 hours reached.
Anticholinergics are also used to control detrusor instability.

172

EMQs by Maju Mathews

Theme: Child neglect


Options
A.
B.
C.
D.
E.
F.
G.

Accidental trauma
Child neglect
Osteogenesis imperfecta
Physical abuse
Malnutrition
Sexual abuse
Normal

For each question below, choose the SINGLE most likely answer form the above list of
options. Each option may be used once, more than once, or not at all.

457. Child with nocturnal enuresis, not gaining weight.


458. A 7 month old infant with spiral fracture or femur. X ray shows absence
of callus formation.
459. An 8-year-old girl with bleeding PV following horse riding. The parents
think it is an accident while riding the horse.
460. A 1-year-old child presents with a scalded burn over his forearm caused
by the fall of a cup of hot tea. He was treated for extensive nappy rash 6
months ago.
461. A 10 days old baby brought to A & E his Caucasian father and AfroCaribbean mother. Material grandmother is worried about the blue
patches over the buttocks.

173

EMQs by Maju Mathews

Answers: Child Neglect


457. (E)
The reason for failure to thrive is secondary to malnutrition especially to
be considered in the light of nocturnal enuresis.

458. (C)
The absence of callus formation on X-ray suggests a non-healing fracture.

459. (F)
It is important to have a high index of suspicion for child neglect especially
in young girls with genitourinary complaints.

460. (B)
Child neglect should be thought of in recurrent unusual presentations esp
Resulting in injury / infection to the child.

461. (G)
This is nothing but Mongolian spots. The parental lineage is misleading
here.

174

EMQs by Maju Mathews

Theme: Bleeding PV
Options
A.
B.
C.
D.
E.
F.

Laparoscopy
Pelvic ultrasound to check placental position
B-HCG estimation
Shift to OT after referral with Gynecologist
Blood group and typing
Kleihauers acid elution test

For each question below, choose the SINGLE most likely answer form the above list of
options. Each option may be used once, more than once, or not at all.

462. Woman with 6 weeks amenorrhoea coming to A&E with acute abdominal
pain and mild bleeding PV. Abdominal examination shows an empty
uterus and no other significant findings. What do you do next?
463. Woman with history of recurrent haemorrhage and still birth in 2 nd trimester
in the previous 3 pregnancies presenting with mild bleeding PV at 28 th
week. What is the next course of action?
464. Patient with history of 10 weeks amenorrhoea (confirmed HCG positive)
presenting with abdominal pain preceded by mild bleeding PV. How would
you proceed next?

175

EMQs by Maju Mathews

Answers: Bleeding PV
462. (B)
Pelvic ultrasound.

463. (F)
Kleihauer test.

464. (D)
Gynecologist referral and shift to OT.

176

EMQs by Maju Mathews

Theme: Management of trauma case


Options
A.
B.
C.
D.
E.
F.
G.
H.
I.

External fixation
Immediate shift to OT
Intravenous fluids
Reduction with splinting
X ray femur
Abdominal Ultrasound
CT-scan head
Blood transfusion
Shift to OT

For each question below, choose the SINGLE most likely answer form the above list of
options. Each option may be used once, more than once, or not at all.

465. Patient coming to A&E with polytrauma. All preliminary resuscitation


measures have been carried out. On examination patient is noticed to
have a deformed thigh. What is the next step in management?
466. Patient coming to A&E with polytrauma. All preliminary resuscitation
measures have been carried out. On examination patient is noticed to
have a deformed thigh. Pulse is 120/min and BP diastolic is 60-mmHg.
What is the initial management step?
467. Patient coming to A&E with polytrauma. All preliminary resuscitation
measures have been carried out. On examination patient is drowsy and
appears confused. Pulse is 60/min and BP is 150/110-mmHg. How do you
proceed next?

177

EMQs by Maju Mathews

Answers: Management of Trauma Case


465. (E)
X-ray femur. Fracture femur is associated with significant hemorrhaging. It
is ruled out by appropriate investigations.

466. (C)
Intravenous fluids. Fracture femur can cause hypovolemia and is likely to
required fluid supplementation.

467. (G)
CT head. Increasing blood pressure with decreasing pulse, also called
Cushings reflex is the result of an increase in intracranial pressure.

178

EMQs by Maju Mathews

Theme: Preventive measure


Options
A.
B.
C.
D.
E.
F.
G.
H.
I.

Cirrhosis
Hepatocellular carcinoma
Infectious mononucleosis
Leptospirosis
Lyme disease
Sclerosing cholangitis
Hepatitis A
Hepatitis B
Hepatitis C

For each question below, choose the SINGLE most likely answer form the above list of
options. Each option may be used once, more than once, or not at all.

468. Careful cleaning and cooking of shellfish.


469. Avoiding swimming in rivers.
470. Immunization of sewage workers.
471. Immunizing health workers in contact with body fluids.
472. Encouraging iv drug users to use needle exchange scheme centres.
473. Immunizing against hepatitis B .. to prevent no only hepatitis B.

179

EMQs by Maju Mathews

Answers: Preventive Measure


468.
469.
470.
471.
472.
473.

180

(G)
(D)
(G)
(H)
(H)
(B)

EMQs by Maju Mathews

Theme: Pain abdominal film abnormalities


Options
A.
B.
C.
D.
E.
F.
G.
H.
I.
J.
K.

Perforated peptic ulcer


Hiatus Hernia
Large bowel obstruction
Chronic pancreatitis
Ulcerative colitis
Infective colitis
Coeliac disease
diverticulitis
small bowel obstruction
Acute pancreatitis
Tropical sprue

For each question below, choose the SINGLE most likely answer form the above list of
options. Each option may be used once, more than once, or not at all.

474. A 50-year-old male presents with gradual central abdominal pain with
copious vomiting. On examination weak pulse at 120/min, rigid abdomen
with generalized tenderness. Pain abdominal X-ray shows absence of
psoas shadow and meteorism sign over the epigastric region.
475. A 45-year-old female, known case of Crohns disease, presents with
colicky abdominal pain preceded by nausea and vomiting. Supine AXR
shows central gas shadows and erect AXR shows air fluid levels.
476. A 35-year-old female presents with fever, weight loss and severe bloody
diarrhoea. On examination weak pulse 120/min, BP 85/55-mmHg and
marked tenderness over lower abdomen with distension. AXR shows wide
colon with loss of normal haustral pattern and absence of feces.
477. A 25-year-old man presents with fever, colicky abdominal pain and profuse
bloody diarrhoea. He gives history of previous such episodes which
subsided spontaneously without treatment. AXR shows air in the colonic
wall.
478. A 38-year-old male presents with marked weight loss and epigastric pain,
which bores through to the back and is relived by local application of heat.
AXR shows speckled calcification is epigastric region. Biochemistry
shows increased blood sugar and normal serum amylase.

181

EMQs by Maju Mathews

Answers: Pain Abdominal Film Abnormalities


474. (J)
Self perpetuating acute inflammation of pancreas. Gradual or sudden
severe central abdominal pain radiating to the back. Sitting forward may
give relief. Causes: Gallstones, alcohol, hyperlipidemia, Polyarteritis
nodosa, azathioprine, and diuretics.
475. (I)
Features: Pain, distension, vomiting & constipation.

476. (E)
Recurrent inflammatory disease of the large intestines always involving
the rectum. Commonest cause of prolonged bloody diarrhoea. Diagnosis:
Sigmoidoscopy, biopsy, barium enema.
477. (F)
Infective colitis caused by gas forming organisms.

478. (D)
Characterized by recurrent abdominal pain, steatorrhea, weight loss and
diabetes mellitus. Pain radiates to the back. Mainly caused by chronic
alcohol abuse.

182

EMQs by Maju Mathews

Theme: Diagnosis of musculoskeletal pain


Options
A.
B.
C.
D.
E.
F.
G.
H.
I.
J.
K.
L.
M.
N.
O.

Malingering
Chondroma
Spondylosis
Spondylolisthesis
Pyogenic infection
Disc prolapse
Stress fracture
Pathological Fracture
Osteoid Osteoma
Myositis ossificans
Scheuermanns disease
Calvs disease
Brodies abscess
Overuse injury
Malignant metastasis

For each question below, choose the SINGLE most likely answer form the above list of
options. Each option may be used once, more than once, or not at all.

479. A 24-year-old man while lifting his luggage develops severe back pain,
which is made worse by straining. He is unable to straighten up with
numbness in the left leg.
480. A 14-year-old girl noted for playing truant from school, otherwise fit
complains of backache and fatigue. Her parents are suspicious and also
notice that the girl is increasingly becoming round shouldered. On
examination movements are normal and a smooth hump in the thoracic
region is seen.
481. Following an injury, a 27-year-old youth gradually developed a tender
swelling near the elbow joint. X-rays show a fluffy density in the soft tissue
near the joint.
482. A 28-year-old man complains of persistent pain in his right lower leg. He is
an avid weight lifter and ignores the pain. He had been advised earlier by
a physiotherapist to exercise in moderation. He also notices some wasting
of the affected area a few days later. On subsequent X-ray examination a
radiolucent area in the right tibia with surrounding sclerosis is seen.
483. A 55-year-old man present with severe diffuse back pain, impotence and
cramps and burning of the lower limbs. He has a long history of back pain
and invasive investigations.

183

EMQs by Maju Mathews

Answers: Diagnosis of Musculoskeletal Pain


479. (F)
Treatment with analgesics and physiotherapy. With extensive neurological
sings and cauda equina syndrome, surgery advocated.

480. (K)
Scheuermanns disease.

481. (J)
Myositis ossificans.

482. (I)
Osteoid osteoma.

483. (O)
Malignant metastasis.

184

EMQs by Maju Mathews

Theme: Diagnosis of lesions around the eye


Options
A.
B.
C.
D.
E.
F.
G.
H.
I.
J.
K.

Malignant melanoma
Dacryoadenitis
Dacryocystitis
Molluscum contagiosum
Viral wart
Basal cell carcinoma
Squamous cell carcinoma
Stye
Chalazion
Xanthelasma
Xanthoma

For each question below, choose the SINGLE most likely answer form the above list of
options. Each option may be used once, more than once, or not at all.

484. A 60-year-old woman presents with a pearly nodular lesion on the right
lower eyelid. A central ulcer is seen.
485. A 25-year-old man with a mildly tender smooth firm mass at the inner
aspect of the lower lid.
486. A 20-year-old youth presents with a painful reddish pimple on lid margin.
487. A 40-year-old man presents with a painful swelling arising acutely in the
superolateral aspect of the right upper lid.
488. A 55-year-old lady with soft yellowish skin lesions around both eyes.
489. A 5-year-old boy with small painless umbilicated papules around the eyes.

185

EMQs by Maju Mathews

Answers: Diagnosis of Lesions around the Eye


484. (F)
Basal cell CA
There are several clinical types of BCC. The most common is
noduloulcerative BCC, which begins as a small, pearly nodule, often with
small telangiectatic vessels on its surface. The nodule grows slowly and
may undergo central ulceration. Various amounts of melanin may be
present in the tumour; tumors with a heavier accumulation are referred to
as pigmented BCC. Superficial BCC consists of one or several
erythematous, scaling plaques that slowly enlarge.
485. (I)
Chalazion (hordeolum internum)
A chalazion is a painless, granulomatous inflammation of a meibomian
gland that produces a peal like nodule within the eyelid. It can be incised
and drained, or injected with glucocorticoids.
486. (H)
Stye (hordeolum externum)
An external hordeolum (sty) is caused by staphylococcal infection of the
superficial accessory glands of Zeis or Moll located in the eyelid margins.
487. (B)
Dacryoadenitis

488. (J)
Xanthelasma
Yellow-colored cutaneous papules or plaques, xanthelasma are found on
the eyelids, whereas tendon xanthomas are frequently associated with the
achillis and extensor finger tendons; The least specific sign for
hyperlipidemia is xanthelasma because at least 50 percent of the patients
with this finding have normal lipid profiles.
489. (D)
Molluscum contagiosum
A benign disease characterized by pearly, flesh-colored, umbilicated skin
lesions 2 to 5 mm in diameter. The infection can be transmitted by closed
contact, including sexual intercourse. Lesions typically occur in the genital
region but can be found anywhere on the body except the palms and the
soles. In most cases the disease is self-limited and has no systemic
complications. Develops especially often in association with the advanced
stages of HIV infection.

186

EMQs by Maju Mathews

Theme: Diagnosis of joint pains


A.
B.
C.
D.
E.
F.
G.
H.
I.
J.
K.
L.

Ankylosing spondylitis
Gouty arthritis
osteoarthritis
Osteochondritis dissecans
Perthes disease
Polymyalgia rheumatica
Pseudogout
Psoriatic arthropathy
Reiters disease
Rheumatoid arthritis
Septic arthritis
Systemic lupus erythematosus

For each question below, choose the SINGLE most likely answer form the above list of
options. Each option may be used once, more than once, or not at all.

490. A 28-year-old male comes with painful knees and ankles with a rash on his
glans penis. He has a history of urethritis due to chlamydia trachomatis.
491. A 22-year-old male develops pain and stiffness of lower back for 6 months.
Examination reveals pain over both the sacroiliac joints.
492. A 69-year-old diabetic lady has fever and pain in right knee for 1 day with
chills and rigors.
493. A 62-year-old lady presents with severe pain stiffness in muscles of
shoulders pelvis for 3 weeks, which is worse in the mornings. ESR is
raised.
494. A 65-year-old woman has an insidious onset of pain affecting the distal
joints of upper limbs symmetrically. On examination, tender
interphalangeal joints with deformity.
495. A 20-year-old football player presents with swollen left knee with locking.
He complains of pain after exercise. Examination shows wasting over the
quadriceps and lateral aspect of the joint.

187

EMQs by Maju Mathews

Answers: Diagnosis of Joint Pains


490. (I)

The triad of arthritis, urethritis, and conjunctivitis, became known as Reiters syndrome, often occurring
with additional mucocutaneous lesions. A similar spectrum of clinical manifestations can be triggered by enteric
infection with any of several Shigella, Salmonella, Yersinia, & Campylobacter species, by genital infection with
Chlamydia trachomatis. Since most patients with spondyloarthropathy do not have the classic features of Reiters
syndrome, it has become customary to employ the terms reactive arthritis. Constitutional symptoms are common,
including fatigue, malaise, fever, & weight loss. The musculoskeletal symptoms are usually acute in onset. Arthritis is
usually asymmetric & additive. The joints of the lower extremities, especially the knee, ankle, & subtalar,
metatarsophalangeal, & toe interphalangeal joints, are the most common sites of involvement, tendinitis & fascitis are
particularly characteristic lesions. Urogenital lesions may occur throughout the course of the disease. In males,
urethritis may be marked or relatively asymptomatic. Prostatitis is also common. Similarly, in females, cervicitis or
salpingitis may be caused. Ocular disease is common, ranging from transient, asymptomatic conjunctivitis to an
aggressive anterior uveitis. Mucocutaneous lesions are frequent. Oral ulcers tend to be superficial, transient, & often
asymptomatic. The characteristic skin, lesions, keratoderma blenorrhagica, consist of vesicles that become
hyperkeratotic, ultimately forming a crust before disappearing. They are most common on the palms & soles but may
occur elsewhere.
491. (A)
The symptoms of the disease are usually first noticed in late adolescence or early adulthood consisting
of low-back morning stiffness which becomes per sistent within months. Arthritis in the hips & shoulders (root joints)
occurs in 25 to 35 percent of patients, in many cases early in the disease course. The most common extraarticular
manifestation is acute anterior uveitis, which can antedate the spondylitis. Aortic insufficiency, sometimes producing
symptoms of congestive heart failure, occurs in a few percent of patients, occasionally early in the course of the spinal
disease. Up to half the patients have inflammation in the colon or ileum. This is usually asymptomatic, but in 5 to 10
percent of patients with AS, frank IBD will develop. The HLA-B27 gene is present in approximately 90 percent of
patients with AS. X-ray shows squaring of the vertebral bodies. Exercise increases mobility & improves function. Most
patients require anti-inflammatory agents to achieve sufficient symptomatic relief to be able to remain functional & carry
out the exercise program.
492. (K)
Ninety percent of patients present with involvement of a single joint: most commonly the knee. The
usual presentation consists of moderate to severe pain that is uniform around the joint, effusion, muscle spasm, &
decreased range of motion. Fever in the range of 38.3 to 38.9 0C (101 to 1020F). A focus of extraarticular infection, such
as a boil or pneumonia, should be sought. Peripheral-blood leukocytosis & a left shift are common findings. Specimens
of peripheral blood & synovial fluid should be obtained before antibiotics are administered. Blood cultures are positive
in up to 50 percent of S. aureus infections but are less frequently positive in infections due to other organism. The
synovial fluid is turbid, serosanguineous, or frankly purulent. Cultures of synovial fluid are positive in more than 90
percent of cases. Prompt administration of systemic antibiotics & drainage of the involved joint can prevent destruction
of cartilage, postinfectious degenerative arthritis, joint instability, or deformity. A third-generation cephalosporin such as
cefotaxime (1 g every 8 h) an oxacillin (2 g every 4 h) will provide adequate coverage for most infections.
493. (F)
The polymyalgia rheumatica syndrome is characterized by stiffness, aching, & pain in the muscles of
the neck, shoulders, lower back, hips, & thighs. In patients with involvement of the temporal artery, headache is the
predominant symptom & may be associated with a tender, thickened, or nodular artery, which may pulsate early in the
disease but become occluded later. Scalp pain & claudication of the jaw & tongue may occur. A well-recognized &
dreaded complication of temporal arteritis, particularly in untreated patients, is ocular involvement due primarily to
ischemic optic neuritis, which may lead to serious visual symptoms, even sudden blindness in some patiens.
Characteristic laboratory findings in addition to the elevated ESR include a normochromic or slightly hypochromic
anaemia. Levels of enzymes indicative of muscle damage such as serum creatine kinase are not elevated. The
diagnosis is confirmed by biopsy of the temporal artery. Since involvement of the vessel may be segmental, the
diagnosis may be missed on routine biopsy; serial sectioning of biopsy specimens is recommended. Temporal arteritis
& its associated symptoms are sensitive to glucocorticoid therapy.
494. (J)
Rheumatoid arthritis RA most often causes symmetric arthritis with characteristic involvement of certain
specific joints such as the proximal in terphalangeal & metacarpophalangeal joints. The distal interphalangeal joints are
realer involved. Synovitis of the wrist joints is a nearly uniform feature of RA & may lead to limitation of motion,
deformity, & median nerve entrapment (carpal tunnel syndrome). Characteristic deformities of the hand include Z
deformity, swan-neck deformity & boutonnire deformity Rheumatoid nodules develop in 20 to 30 percent of persons
with Ra. Clinical weakness & atrophy of skeletal muscle are common. Rheumatoid vasculitis can cause polyneuropathy
& mononeuritis multiplex, cutaneous ulceration & dermal necrosis, digital gangrene, & visceral infarction.
Pleuropulmonary manifestations, which are more commonly observed in men, include pleural disease, interstitial
fibrosis, pleuropulmonary nodules, pneumonitis, & arteritis. Rheumatoid factors, which are autoantibodies reactive with
the Fc portion of 1gG, are found in more than two-thirds of adults with the disease. Widely utilized test largely detec
IgM rheumatoid factors. The presence of rheumatoid factor does not establish the diagnosis of RA but can be of
prognostic significance. The erythrocyte sedimentation rate is increased in nearly all patients with active RA. Juxtaarticular osteopenia may become apparent within weeks of onset. Loss of articular cartilage & bone erosion develop
after months of sustained activity.
495. (D) Osteochondritis dissecans.

188

EMQs by Maju Mathews

Theme: Diagnosis of pediatric emergencies


Options
A.
B.
C.
D.
E.
F.
G.
H.
I.
J.
K.
L.

Metabolic encephalopathy
Brain herniation
Raised intracranial pressure
Heat stroke
Reyes syndrome
Hypovolemic shock
Septicemic shock
Cardiogenic shock
Cardiac arrest
Drug toxicity
Hypoglycemia
Toxic shock syndrome

For each question below, choose the SINGLE most likely answer form the above list of options.
Each option may be used once, more than once, or not at all.

496. A 12-year-old school girl has high fever with chills and sore throat. She has been
passing loose watery stools. On examination, temperature is 38.7 0C. pulse is
130/min and BP is 90/60-mmHg. A generalized desquamative rash is noted all
over the body.
497. An 8 months old infant is seen with high fever, chills and rigors of 8 hours
duration. On examination child is irritable, crying excessively and is progressively
getting drowsy. A pink macular rash is noticed all over the trunk and limbs.
498. A 5-year-old boy was observed by his mother to have a poor appetite since 2
weeks. He has since then complained of abdominal pains and vomited on
several occasions. He progressively became drowsy and confused, slept
throughout the day and stayed awake at nights. On examination, he was drowsy
and jaundiced. A flapping tremor was demonstrable.
499.

A 12-year-old girl on summer holidays with her family in Cyprus suddenly


collapses in the beach. She is weak and drowsy. She is admitted in a local
hospital. On assessment, her temperature is 38.8oC, pulse of 100/min and BP is
106/70-mmHg. She is given antipyretics and started on dextrose infusions. Her
condition deteriorates and she becomes comatose with renal shutdown. Pulse is
now 140/min and BP has fallen to 74/40-mmHg. On catheterization around
100ml of dark brown urine is collected.

500. A 2-year-old girl develops fever for which she is given crushed aspirin tablets by
her mother. The fever persists and she becomes drowsy and hypotonic and then
lapses into coma.

189

EMQs by Maju Mathews

Answers: Diagnosis of Pediatric Emergencies


496. (L)
A severe septicemic illness with shock and a desquamative rash in a
teenage girl is highly suggestive of toxic shock syndrome from tampons.

497. (G)
Meningococcal septicemia and septicemic shock. Always exclude
meningococcus in any child with fever and rash. This is because it kills
quickly and delay in commencing penicillin can be fatal.

498. (A)
This child is developing liver failure. Reversal of sleep rhythm and flapping
tremor points to a liver or renal failure. The presence of jaundice narrows
it to the liver.

499. (D)
This is heat stroke. The deterioration in her clinical state overnight was
contributed to by the use of dextrose infusions alone rather than dextrose
and saline. The brown urine is due to myoglobinuria and will worsen the
hypovolemic renal failure. She is best managed in an ITU setting.

500. (E)
Fever, hypotonia and coma in a child of this age, who has used aspirin
should make you think of Reyes syndrome.

190

EMQs by Maju Mathews

Theme: Investigation of abdominal pain in childhood


Options
A.
B.
C.
D.
E.
F.
G.
H.
I.
J.
K.
L.
M.
N.

Plain X-ray of the abdomen


HIV screening
Genotype
Upper gastrointestinal endoscopy
Factor VIII screen
No investigations required
Psychological evaluation
Electrolytes and urea and creatinine
Examination of urinary sediments
Blood sugar
Chest X-ray
Serum lead estimation
Stool for occult blood
Abdominal ultrasound

For each question below, choose the SINGLE most likely answer form the above list of
options. Each option may be used once, more than once, or not at all.

501. Eight-month old infant with episodic crying, pulling up of the legs, vomiting
and the passage of bright red blood per rectum. A sausage shaped mass
is palpable per abdomen.
502. A mother, worried that her six-month old child is not growing, brings her to
the outpatients. She has been treated twice for pneumonia. She has been
regurgitating her feeds with distress and turns blue after feeding.
503. A 3year old Jamaican child has been having recurrent painful episodes in
the abdomen and limbs. On examination, she is mildly pale and jaundiced
and diffusely tender all over the abdomen.
504. An 8-year-old boy complains of fever with chills and rigors, cough with
mucoid sputum, and right sided upper abdominal pains. There is no
vomiting or diarrhoea. On examination, he is toxic, and jaundiced. There
is vague tenderness and mild guarding in the right hypochondrium.
505. A 7-year-old boy has been complaining of repeated episodes of abdominal
pains. Parents have newly migrated to UK from South Africa. Appendicitis,
colitis and helminthiasis have been queried on different occasions. He has
had frequent hospital admissions and extensive investigations and as a
result, he has been unable to settle down in his new school. The pain is
located in the left iliac fossa and radiates to the right hypochondrium.
Abdominal examination; intensely tender in all areas but there is no
guarding or rebound. Rest of the systems are normal.

191

EMQs by Maju Mathews

Answers: Investigation of Abdominal Pain in Childhood


501. (A)
The infant has intussusception. Barium enema would be the first choice
but since it isnt in the answers an x ray would be the most appropriate
choice claw sign.

502. (D)
Upper GI endoscopy is
tracheoesophageal fistula.

indicated

for

esophageal

atresia

and

503. (C)
Genotype for sickle cell disease-other causes of medical acute abdomen
eg. diabetes, Henoch-Schnlein purpura, acute intermittent porphyria,
scleroderma etc.

504. (K)
Pneumonia may present with jaundice and a septicemic illness.

505. (G)
This boy has recently moved to a new environment, a new school, and is
now presenting with a pattern of abdominal pain that does not conform to
a definite pathology. Each episode allows him time away from school. Is
he unhappy, perhaps malingering. He should have a psychological
evaluation. There are no fevers so Familial Mediterranean fever is not
likely.

192

EMQs by Maju Mathews

Theme: Investigation of abdominal pain in childhood


Options
A.
B.
C.
D.
E.
F.
G.
H.
I.
J.
K.
L.
M.
N.

Plain X-ray of the abdomen


HIV screening
Genotype
Upper gastrointestinal endoscopy
Factor VIII screen
No investigations required
Psychological evaluation
Electrolytes and urea and creatinine
Examination of urinary sediments
Blood sugar
Chest X-ray
Serum lead estimation
Stool for occult blood
Abdominal ultrasound

For each question below, choose the SINGLE most likely answer form the above list of
options. Each option may be used once, more than once, or not at all.

506. A 12-year-old boy is woken up regularly at night by a gnawing epigastric


pain, which is only relieved by eating. Except for epigastric tenderness,
examination of the abdomen and rest of the systems are normal.
507. A 10-year-old hemophiliac complains of a sudden onset of right-sided
upper abdominal pain, which radiates to the back on the right. This is
followed two days later by a linear copy of vesicles over the site of the
pain. The rest of the abdomen is not tender.
508. A 5-year-old boy has fever, cough and running nose. Three days after this,
he develops a several abdominal pain. His mother notices a purples rash
over the buttocks and legs. On closer look at the rashes is observed that
they do not fade on pressure.
509. A 7-year-old boy complains of colicky abdominal pain. His parents report
that he has been acting strangely latterly, eating objects like paper and
clothes.
510. A 12-year-old girl reports to her mother that she has been having cramplike lower abdominal pains, nausea and vomiting. This is followed the next
day by the passage of blood per vagina. A similar sequence of pain and
bleeding occurred five weeks ago and she is frightened that she may be
dying.

193

EMQs by Maju Mathews

Answers: Investigation of abdominal pain in childhood


506. (D)
Peptic ulcer; upper GI endoscopy.

507. (B)
Hemophiliac, blood transfusion, HIV, herpes zoster. Pain comes before
the rash so may be mistaken for an acute abdomen.

508. (I)
Henoch Schnlein purpura.

509. (L)
Lead toxicity.

510. (F)
She has just attained menarche. Reassurance and a talk will help.

194

EMQs by Maju Mathews

Theme: Management in epidemiological cases


Options
A.
B.
C.
D.
E.
F.
G.

Barrier nursing
Contact racing
Quarantine
Refer to infectious disease consultant
Vaccination with Immunoglobulin
Isolation
Prophylaxis with Rifampicin

For each question below, choose the SINGLE most likely answer form the above list of
options. Each option may be used once, more than once, or not at all.

511. A girl develops diarrhea after returning from a holiday to India. She had
been to her grand father, who owns a restaurant there.
512. A pregnant lady says that she had a contact with a child who was
diagnosed of suffering from rubella.
513. A child with fever develops erythematous maculopapular vascular rash.
514. A boys friend diagnosed of having meningococcal infection, mother is
worried about her son getting the infection.

195

EMQs by Maju Mathews

Answers: Management in Epidemiological Cases


511. (D)

512. (E)

513. (F)
Suspected measles infection for which isolation prevents risk of
transmission to others.

514. (G)

196

EMQs by Maju Mathews

Theme: Abnormal movements and movement disorders


A.
B.
C.
D.
E.
F.
G.
H.
I.
J.
K.
L.

Sydenhams Chorea
Hemiballismus
Tics
Myoclonus
Dystonia
Wilsons Disease
Athetosis
Parkinsonism
Tardive dyskinesia
Gilles de la Tourettes syndrome
Huntingtons chorea
Akathisia

For each question below, choose the SINGLE most likely answer form the above list of
options. Each option may be used once, more than once, or not at all.

515. A 14-year-old boy presenting with dysarthria, tremor, dysphagia and


mental retardation. On examination has slow writhing movements of his
left arm and leg.
516. An 18-year-old has frequent involuntary contractions of facial muscles,
which make him wink or pout. He also has occasional aggressive verbal
outbursts with searing.
517. A 45-year-old male presents with jerky, purposeless hand movements. His
wife reports that he has become increasingly irritable and forgetful of late.
His late uncle was also similarly affected.
518. A 60-year-old diabetic complains of involuntary movements of his left arm
and leg. O/E, he is seen to make sudden violent kicking movements.
519. A 30-year-old schizophrenic male on antipsychotic medication is brought
to A&E with restlessness and agitation. During overnight observation in
the ward he is found to be frequently getting in and out of his bed with
occasional pacing about in the room.
520. A 55-year-old woman on treatment for chronic schizophrenia complains of
continuous involuntary movements of her mouth and tongues. On
examination noticed to make frequent grimacing and chewing
movements.

197

EMQs by Maju Mathews

Answers: Abnormal Movements And Movement Disorders


515. (F)
Also known as hepatolenticular degeneration. Due to deposition of copper
in liver brain & basal ganglia.

516. (J)
Usually begins in childhood and can present with wide varicty of
symptoms.

517. (K)
Autosomal dominant. Insidious onset. Rapidly progressive to death.

518. (B)
Recognized association with Diabetes.

519. (L)
Side effects of neuroleptics. Can drive the patient to suicide in rare
instances.

520. (I)
Adverse affects of neuroleptics. Treated with Tetrabenazine.

198

EMQs by Maju Mathews

Theme: Common or important adverse effect of drugs


A.
B.
C.
D.
E.
F.
G.
H.
I.
J.
K.
L.

Amiodarone
Amitriptyline
Atenolol
Captopril
Carbimazole
Chlorpromazine
Indomethacin
Lithium carbonate
Nifedipine
Prednisolone
Simvastatin
Sulfasalazine

For each question below, choose the SINGLE most likely answer form the above list of
options. Each option may be used once, more than once, or not at all.

521. A patient has sustained a Colles fracture. For some time she has noticed
weight gain, thirst and mood swings.
522. A patient has become prone to sunburn even after moderate exposure to
the sun. He notices a grayish discoloration of his skin.
523. A patient complains of wheezing on exertion. He has also become
impotent.
524. A patient complains of painful, swollen gums that bleed on brushing.
525. A patient develops a sore throat and is found to be neutropenic.
526. A patient develops painful, tender shoulder and thigh muscles. His
creatine kinase is markedly elevated.

199

EMQs by Maju Mathews

Answers: Common Or Important Adverse Effect of Drugs


521. (J)
Iatrogenic clsohinge syndrome.
522. (A)
Causes photosensitivity.
523. (C)
B-blockers induced impotence.
524. (I)
Given hyerestrophy also called by phenytoin.
525. (E)
Always alert & patient on cartinazole about the warming signs of
neutropenia like fever, sore threat.
526. (K)
Myotis is a lewgnied application of station dryes.

200

EMQs by Maju Mathews

Theme: Choice of radiological investigations


Options
A.
B.
C.
D.
E.
F.
G.
H.
I.
J.
K.
L.
M.

Barium meal
Barium enema
Barium swallow
Ultrasound
CT scan
MRI
SPECT scan
HIDA scan
Intra venous urogram
Tcm DTPA scan
Tcm DMSA scan
Serum ferritin
Liver biopsy

For each question below, choose the SINGLE most likely answer form the above list of
options. Each option may be used once, more than once, or not at all.

527. Assessment of vascular deficit in stroke.


528. To establish a diagnosis of hiatus hernia.
529. To confirm presence of ectopic pregnancy.
530. To diagnose atresia of biliary ducts.
531. Detection of urinary bladder pathology.
532. To study uptake and excretory pattern of each kidney i.e. renal function.
533. To diagnose hemochromatosis.

201

EMQs by Maju Mathews

Answers: Choice of Radiological Investigations


527.
528.
529.
530.
531.
532.
533.

202

(G)
(C)
(D)
(H)
(D)
(J)
(M)

EMQs by Maju Mathews

Theme: Epidemiology
A.
B.
C.
D.
E.
F.
G.
H.
I.
J.
K.

PSA screening
Mammography
High sensitivity test
High specificity test
Case control study
Cohort study
Double blind study
Gaussian distribution
Skewed distribution
Causal relationship
Evidence based practice

For each question below, choose the SINGLE most likely answer form the above list of
options. Each option may be used once, more than once, or not at all.

534. Reliably detects subjects having the disease.


535. An example of an effective screening test.
536. Study which generates incidence data.
537. Demonstration of dose-response relationship is a good evidence of this.
538. A distribution where 5% of subjects lie outside the normal distribution
curve.
539. Streptokinase decreases mortality in MI patients. The above was
concluded after studies of numerous researches and is an example of..

203

EMQs by Maju Mathews

Answers: Epidemiology
534.
535.
536.
537.
538.
539.

204

(C)
(B)
(F)
(J)
(H)
(K)

EMQs by Maju Mathews

Theme: Antidotes in poisoning


Options
A.
B.
C.
D.
E.
F.
G.
H.
I.
J.
K.
L.
M.

Flumazenil
Pralidoxime
Dicobalt edetate
Digoxin specific antibody
Phytomenadione
Dimercaprol
Procyclidine
Iv Acetylcysteine
Naloxone
Atropine
Antitoxin
Glucagon
No specific antidote

For each question below, choose the SINGLE most likely answer form the above list of
options. Each option may be used once, more than once, or not at all.

540. A 23-year-old student presents with diplopia, blurred vision, photophobia


and paralysis. His cardiorespiratory status is severely compromised. He
does not give history of any overdose but says that the previous night he
had canned tuna.
541. A 65-year-old man, on antihypertensive medication, brought to A&E with
collapse. He is living alone and was on discontinued treatment for
depression. On examination pulse rate is 36/min and BP is 65/40-mmHg.
His pulse rate does not pick up following administration of atropine.
542. A 28-year-old woman brought to A&E with fits and shock. She has a
characteristic breath of bitter almonds.
543. A 30-year-old male with chronic schizophrenia, on neuroleptic medication,
presents to A&E with torticollis, opisthotonus and glossopharyngeal
dystonia.
544. A 65-year-old female brought to A&E with sweating and respiratory
distress She also complains of salivation, lacrimation, urination and
diarrhoea.
545. A 32-year-old woman presents with respirator arrest and severe
constipation. On examination she has pinpoint pupils with signs of
pulmonary fluid overload.

205

EMQs by Maju Mathews

Answers: Antidotes in Poisoning


540. (K)
Characteristic of botulinum poisoning. Absence of GI symptoms.
541. (L)
Overdose of B-blockers. Glucagon given if atropine fails.
542. (C)
Cyanide poisoning.
543. (G)
Generally responds to Procyclidine.
544. (J)
Cholinergic symptoms probably from OP poisoning.
545. (I)
Given until breathing adequate.

206

EMQs by Maju Mathews

Theme: Diagnosis of skin lesions


A.
B.
C.
D.
E.
F.
G.
H.
I.
J.
K.
L.
M.

Pemphigus vulgaris
Pyoderma gangrenosum
Toxic epidermal necrolysis
Pretibial myxoedema
Bullous pemphigoid
Erythema multiforme
Dermatitis herpetiformis
Psoriasis
Erythema nodosum
Erythema marginatum
Erythema induratum
Seborrheic dermatitis
Secondary syphilis

For each question below, choose the SINGLE most likely answer form the above list of
options. Each option may be used once, more than once, or not at all.

546. A 30-year-old lady developed painful blisters all over her body and buccal
mucosa with painful micturition and defaecation. She is a known case of
epilepsy on treatment with sodium valproate.
547. A 13year old girl present with an ulcerated rash over her leg. Biopsy done
shows localized areas of fat necrosis.
548. A 17-year-old sexually active girl presents with painful, raised red lesions
in the front of her shins and forearms. She uses the combined pill for
contraceptive cover.
549. A 65-year-old woman presents with 4 months history of tense blisters
forming over the trunk and lower limbs which was preceded by itching and
burning sensation. There are no lesions in the mouth.
550.

A 40-year-old man presents with a 6-month history of pruritus.


Examination revealed widespread excoriations with blisters, most of which
are ruptured. He is found to have anti-endomysial antibodies.

551. A 35-year-old woman has noticed reddish elevated skin lesions with an
edematous appearance over her lower legs. Examination revealed
exophthalmos.

207

EMQs by Maju Mathews

Answers: Diagnosis of Skin Lesions


546. (F)
Erythema multiforme is an acute, self-limited inflammatory disorder of skin mucous
membranes characterized by distinctive iris or target lesions, usually acrally distributed &
often associated with sore throat & malaise. Many drugs, including sulfonamides,
penicillin, phenytoin, & phenylbutazone, have been reported to cause. Infectious agents
associated with EM include Mycoplasma pneumoniae, Histoplasma capsulatum,
Coccidioides immitis, Yersinia enterocolitica, & several viruses. In young adults,
infections with herpes simplex are the most common cause of EM. Stevens-Johnson
syndrome (SJS) is a blistering disorder that is usually more severe than erythema
multiforme. Fever & malaise also occur.
547. (K)
Erythema induratum/nodular vasculitis is an idiopathic disease; however, approximately
25 to 50 percent of patients, polymerase chain reaction (PCR) analysis will demonstrate
Mycobacterium tuberculosis complex DNA. The lesions of lupus profundus are found
primarily on the face, upper arms, & buttocks (sites of abundant fat) & are seen in both
the cutaneous & systemic forms of lupus.
548. (I)
The most common associations are streptococcal infections, upper respiratory infections,
sarcoidosis, & inflammatory bowel disease. The less common associations include
tuberculosis, histoplasmosis, coccidioidomycosis psittacosis, drugs (oral contraceptives,
sulfonamides, aspartame, bromides, iodides), cat-scratch fever, & infections with
Yersinia, Salmonella, & Chlamydia.
549. (E)
Bullous pemphigoid (BP) is a subepidermal blistering skin disease usually seen in the
elderly. Lesions typically consist of tense blisters on either normal-appearing or
erythematous skin. The lesions are usually distributed over the lower abdomen, groin, &
flexor surface of the extremities; oral mucosal lesions are found in 10 to 40 percent of
patients (as compared to 90% of patients with or mucosal lesions in pemphigus vulgaris).
Pruritus may be nonexistent or severe. As lesions evolve, tense blisters tend to rupture &
be replaced by flaccid lesions or erosions with or without surmounting crust. Non
traumatized blisters heal without scarring. Patients with BP do not have an increased
incidence of malignancy.
550. (G)
Dermatitis herpetiformis (DH) is an intensely pruritic, chronic papulovesicular skin
disease characterized by lesions symmetrically distributed over extensor surfaces (i.e.
elbows, knees, buttocks, back, scalp, & posterior neck). Because pruritus is prominent,
patients may present with excoriations & crusted papules but no observable primary
lesions. Almost all DH patients have an associated, usually subclinical, gluten-sensitive
enteropathy, & more than 90 percent express the HLA-B8/DRw3 & HLA-DQw2
haplotypes.
551. (D)
The dermopathy usually occurs over the dorsum of the legs or feet & is termed localized
or pretibial myxedema. It is usually a late phenomenon but is not a manifestation of
hypothyroidism. About half of cases occur during the active stage of thyrotoxicosis, &
virtually all have ophthalmopathy. The affected area is usually demarcated from normal
skin by being raised & thickened & having a peau-d-orange appearance; it may also be
pruritic & hyperpigmented. The most common presentation is a nonpitting edema, but
the lesions may be plaque like, nodular, or polypoid in configuration.

208

EMQs by Maju Mathews

Theme: diagnosis of shock & collapse in children


Options
A.
B.
C.
D.
E.
F.
G.
H.
I.
J.
K.
L.

Metabolic encephalopathy
Brain herniation
Raised intracranial pressure
Heat stroke
Reyes syndrome
Hypovolemic shock
Septicemic shock
Cardiogenic shock
Cardiac arrest
Drug toxicity
Hypoglycaemia
Toxic shock syndrome

For each question below, choose the SINGLE most likely answer form the above list of
options. Each option may be used once, more than once, or not at all.
552. An eight-year-old boy is discovered unconscious in bed in the morning. He lives
alone with his divorced unemployed mother. She suffers from depression for
which she takes tricyclic antidepressants. She had spent the night partying with
friends, leaving the boy home alone. On examination, he is comatose and
sweaty, his pulse is irregular and pupils are dialed.
553. An unemployed single parent brings her seven-month-old infant to the
emergency. She complains that the child has been irritable and drowsy and has
been vomiting. She denies any history of trauma. On examination, there is a
right sided subconjunctival hemorrhage and right-sided papillary dilatation. Pulse
is 50 beats per minutes and there is papilledema on fundoscopy.
554. A 12-year-old boy is receiving doxorubicin for a rhabdomyosarcoma. On followup in the clinic, he is observed to be weak and drowsy. He is not pale. His
extremities are cold and clammy and the pulse is barely palpable. Jugular
venous pressure is elevated. Blood pressure is 70/50-mmHg. Apex beat is not
palpable and heart sounds appear distant.
555. A nine-year old girl complains of excessive urination and thirst. She has
progressively become weaker and drowsy. On examination, she is severely
dehydrated with a furred-coated tongue. The respiration is rapid and deep.
556. A ten-year old boy has sore throat, catarrh and a milk fever. A few days later he
complains of palpitations and weakness. ON examination, his extremities are
cold an clammy. Temperature is 37.2oC and pulse rate is 140 beats per minute,
with several missed beats. A 3rd heart sound is audible at the apex.

209

EMQs by Maju Mathews

Answers: Diagnosis of Shock and Collapse in Children


552. (J)
Child of depressed mother on antidepressants. Child was left alone and
has ingested the mothers tablets. Anticholinergic effects (dilated pupils)
and cardiac arrhythmias are the threats to life.
553. (C)
This child has raised intracranial pressure. This could have resulted either
from neglect (child falling off a couch and hitting the head) or from
physical abuse (excessive shaking injury). Look for other evidence of
physical abuse or neglect e.g. unusual fractures, multiple fractures,
growth failure, and attitude of the child to the mother. All these must be
documented in your notes.
554. (H)
Doxorubicin is a well-known cause of congestive cardiomyopathy and
cardiogenic shock.

555. (F)
This is DKA and should be regarded as a hypovolemic shock rather than
as metabolic encephalopathy because hypovolemia is the threat to life.
Treatment consists of urgent infusion of saline and low dose insuline.

556. (H)
Cardiogenic shock preceded by upper respirator tract infection. Viral
myocarditis is likely. Note the temperature pulse dissociation i.e.
tachycardia out of proportion to fever. This is classical in carditis.
Rheumatic fevers should also be considered but is less likely since the
carditis in rheumatic fever seldom proceeds to severe shock.

210

Você também pode gostar